Ketaksamaan
Ajat Adriansyah August 31, 2006
ii
Pengantar Tulisan ini lebih banyak menekankan pada materi, untuk lebih mahir dalam pengerjaan soal, pembaca diharapkan untuk mencari soal-soal sebanyak mungkin untuk dikerjakan. Apabila terdapat kesalahan dalam penulisan buku ini, harap dilaporkan ke
[email protected]. Ucapan terima kasih untuk: Fajar Yuliawan, Dimas Yusuf W, Aleams barra, dan Albert Gunawan.
Daftar isi 1 Ketaks Ketaksama amaan an Rataan Rataan 1.1 Ketaks Ketaksama amaan an dasar dasar . . . . . . . . . . . . . . . . . . . 1.2 * Pasanga Pasangan n terurut terurut dan Perm Permutasi utasi . . . . . . . . . . 1.3 Ketaksama Ketaksamaan an Rataan Rataan Aritmatik Aritmatik - Rataan Rataan Geometr Geometrik ik 1.4 Ketaksama Ketaksamaan an Akar Akar Rataan Rataan -AM -AM . . . . . . . . . . . 1.5 Ketaks Ketaksama amaan an Jensen Jensen . . . . . . . . . . . . . . . . . .
. . . . .
. . . . .
. . . . .
. . . . .
. . . . .
. . . . .
. . . . .
1 1 4 5 12 15
2 Ketaks Ketaksama amaan an Norm Norm 25 2.1 Ketaksama Ketaksamaan an CauchyCauchy-Sch Schwarz warz (CS), (CS), Minkowski Minkowski dan H¨older older . . 25 2.2 Rearrangem Rearrangement ent & Ketaksam Ketaksamaan aan Chebishe Chebishev v . . . . . . . . . . . . 34 3 Penju Penjumla mlahan han Simetr Simetrik ik dan Siklis Siklis 47 3.1 Normalisasi Normalisasi dan Subtitusi Subtitusi . . . . . . . . . . . . . . . . . . . . . . 47 3.2 Schu Schurr & Muir Muirhea head d . . . . . . . . . . . . . . . . . . . . . . . . . . 55 4 Mixing Mixing Vari Variabel abel dan Metode Metode Kalkulu Kalkuluss 63 4.1 4.1 Refe Refere rens nsii . . . . . . . . . . . . . . . . . . . . . . . . . . . . . . . 71
iii
iv
DAFTAR ISI
Bab 1
Ketaksamaan Rataan 1.1 1.1
Keta Ketaks ksam amaa aan n da dasa sarr
Ketaksamaan yang akan diperkenalkan pertama kali ini merupakan salah satu jenis ketaksamaan yang paling dasar dan sering disebut trivial atau cuma cuma yaitu bahwa kuadrat suatu bilangan rill selalu lebih besar dari nol. Secara matematis ditulis untuk setiap x bilangan riil berlaku x2 0, Kesamaan terjadi jika dan hanya jika x = 0. Ketaks Ketaksama amaan an ini akan akan sangat sangat mem memban bantu tu dalam membuktikan ketaksamaan lain.
−
≥
Contoh 1.1.1 Tentukan Nilai minimum dari x2 imum tersebut tercapai!
− 12 12x x + 6, kapankah nilai min-
Dengan memodifikasi sedikit x2 12 12x x + 6 , akan akan diperoleh bahwa bahwa x2 12 12x x+6 = 2 2 x 12 12x x + 36 30 = (x (x 6) 30 30. Jadi nilai minimum minimum dari fungsi diatas adalah adalah -30. Nilai tersebut tersebut akan akan tercapai jika x 6 = 0 atau x = 6 .
−
−
−
−
− − ≥−
−
Contoh 1.1.2 Buktikan Buktikan ketaksamaa ketaksamaan n nilai tengah aritmatik aritmatik - geometr geometrik ik (AMGM) untuk dua variabel yaitu a+b 2
∀ a, b ≥ 0,
≥
√
ab
dengan kasamaan tercapai untuk a = b .
√ −√ ≥
Solusi Berdasarkan ketaksamaan trivial ( a b)2 0, jadi a + b a+b 0 ab. ab. Jika a = b maka kita peroleh kesamaan, jika 2
⇒
≥
√
a+b = 2 maka a2
√
ab
− 2ab + b2 = 0 dan diperoleh a = b. 1
−2
√
ab
≥
2
BAB 1. KETAKSAMA KETAKSAMAAN AN RAT RATAAN
Contoh 1.1.3 Buktikan ketaksamaan nilai tengah aritmatik - geometrik (AMGM) untuk tiga variabel yaitu
∀ a, b ≥ 0,
a+b+c 3
≥
√ 3
abc
dengan kasamaan tercapai untuk a = b = c . Solusi Untuk setiap a,b,c bilangan riil positif berlaku 1 (x + y + z )[(x )[(x 2
− y)2 + (x (x − z )2 + (y (y − z )2 ] ≥ 0
yang akan terjadi samadengan jika dan hanya jika x = y = z, dengan ekspansi kita perole p eroleh h x3 + y 3 + z 3 3xyz
≥ √ √ √ Subtitusikan x = a , y = b dan z = c kita perole p eroleh h √ a+b+c ≥ abc. 3 3
3
3
3
Ketaksamaan trivial tampak begitu sederhana, tapi ketaksamaan ini merupakan induk dari ketaksamaan lain. Beberapa ketaksamaan yang akan dibahas di Bab berikutnya melibatkan ketaksamaan trivial pada penurunannya. Bahkan banyak soal yang sangat susah yang gagal diselesaikan dengan metode - metode pada bab berikutnya tapi dapat diselesaikan dengan mengubahnya ke bentuk (E (a,b,c)) a,b,c))2 0. Walaupun diperkenalkan lebih awal, bukan berarti ketaksamaan ini adalah yang yang paling mudah digunak digunakan, an, kemampua kemampuan n kita dalam menggunaka menggunakan n identiidentitas sangat berpengaruh dalam menyelesaikan soal pertidaksamaan dengan cara mengubahnya menjadi bentuk kuadrat. Berikut ini diberikan sebuah contoh soal dari seleksi tim olimpiade Cina 2005, yang juga muncul pada [ONI]1 beserta solusinya yang begitu elegan untuk soal sesulit ini. Soal ini termasuk termasuk soal susah bahkan bagi yang sudah mempela jari materi-materi pada subbab berikut.
≥
Contoh 1.1.4 (China 2005 , Vasile Cirtoaje) Diberikn bilangan riil positif a,b,c,d dengan abcd = 1. Buktikan 1 1 1 1 + + + 2 2 2 (a + 1) (b + 1) (c + 1) (d + 1)2
≥1
Solusi Pertama - tama kita mulai dengan membuktikan lemmma berikut Lemma Jika x dan y adalah bilangan riil positif, maka berlaku 1 1 + 2 (1 + x) (1 + y )2 1
≥ 1 +1 xy
Old and New Inequalities oleh Vasile Cirtoaje, Mircea Lascu, Titu Andrescu dan Gabriel Dospinescu. Gil Publisher
3
1.1. KETAKSAMA KETAKSAMAAN AN DASAR DASAR Bukti Lemma Perhatikan bahwa 1 1 + 2 (1 + x) (1 + y)2
xy(x − y)2 + (xy (xy − 1)2 − 1 +1 xy = (1xy( ≥0 + x)2 (1 + y 2 )(1 + xy) xy )
Jadi lemma telah terbukti. Dengan Lemma tersebut kita dapatkan 1 1 + 2 (1 + a) (1 + b)2
≥ 1 +1 ab
dan
1 1 + 2 (1 + c) (1 + d)2
≥ 1 +1 cd
Jumlahkan kedua ketaksamaan kita dapatkan 1 1 1 1 + + + 2 2 2 (a + 1) (b + 1) (c + 1) (d + 1)2
1 1 + 1 + ab 1 + cd 1 + ab + cd + 1 = 1 + ab + cd + abcd = 1
≥
karena abcd = 1. Kebanyakan orang mengatakan suatu soal pertidaksamaan itu mudah setelah mereka melihat solusinya, tapi perlu ditekankan bahwa mencari sebuah solusi jauh lebih sulit daripada memahaminya. Solusi dari contoh diatas walaupun singkat dan terlihat sederhana tapi sulit untuk ditemukan. ditemukan. Ketaksamaan pada lemma yang merupakan kunci dari solusi diatas tidak mudah untuk terpikirkan. Tapi para pembaca pembaca tidak harus terpaku terpaku pada satu solusi solusi dari satu soal, sebuah masalah dapat mempunyai banyak solusi dan sangat penting bagi kita untuk mempelajari mempelajari atau menemuk menemukan an solusi lain dari suatu masalah masalah yang telah diselesaikan. Oleh karena itu akan banyak penyelesaian contoh - contoh pada buku ini diberikan diberikan lebih dari satu solusi, walaupun walaupun b eberapa eberapa diantaran diantaranya ya lebih panjang panjang dan rumit tapi diaharapkan dapat menambah kreativitas berpikir. Ketaksamaan berikutnya merupakan suatu jenis ketaksamaan geometri nontrivial yang pertama yaitu ketaksamaan segitiga yang menyatakan jumlah pan jang dua sisi segitiga lebih besar dari sisinya yang lain. Teorema 1.1.1 Jika x dan y adalah dua buah bilangan riil maka berlaku
|x + y| ≤ |x| + |y| Hal ini juga mengidentifikasikan bahwa pada segitiga dengan panjang sisi a, b dan c berlaku a+b>c a+c>b b+c >a Ditambah lagi jika a,b,c adalah panjang sisi-sisi segitiga, maka dapat menggunakan subtitusi a = x + y, b = y + z dan c = x + z untuk bilangan riil x, y , z . Subtitusi Subtitusi ini dinamak dinamakan an subtitusi Ravi yang diambil diambil dari nama Ravi seorang kontestan Olimpiade yang pertama kali menggunakannya menggunakannya pada kompetisi.
4
BAB 1. KETAKSAMA KETAKSAMAAN AN RAT RATAAN
Latihan 1
1. Berapak Berapakah ah nilai minimum minimum dari cos 2θ 13 ! 2 2. Buktikan 2(x 2(x + y2 ) (x + y )2 untuk x, y bilangan bilangan riil positif. 3. Jika a, b, c, d adalah bilangan riil berapakah nilai minimum dari
−
≥
a2 + b2 + c2 + d2 + e2 = 2a
− 7b + 4c 4c + 3d 3d − 11 11ee + 23
4. Jika a , b dan c adalah bilangan riil positif yang memenuhi a + b + c = 3. Buktikan ab bc ac 3 + + a2 + 1 b2 + 1 c2 + 1 2 5. (Albert Gunawan) Jika a,b,c adalah bilangan riil positif yang mememnuhi ab + ac + bc = 12, buktikan
≤
abc(3 abc(3a a + b + c)(a )(a + 3b 3b + c)(a )(a + b + 3c 3 c)
≥ (12a (12a − bc)(12 bc)(12bb − ac)(12 ac)(12cc − ab) ab) Petunjuk: Buktikan bahwa 3a 3a2 + ab + ac ≥ 12 12a a − bc. bc.
6. Untuk setiap a,b,c bilangan riil buktikan bahwa (a2 + b2 )2 + (a (a2 + c2 )2 + (b (b2 + c2 )2
1.2
≥ 43 (a3 + b3 + c3)(a )(a + b + c)
* Pasa Pasanga ngan n teru terurut rut dan Permut ermutasi asi
Pada subbab ini akan diperkenalkan sebuah konsep dasar dalam matematika yaitu pasangan terurut, tujuannya adalah untuk memudahkan penuliskan solusi - solusi solusi kita dengan dengan notasi matematik matematika a yang umum. Bagi para pembaca pembaca yang yang sudah familiar dengan subjek ini dapat segera melanjutka melanjutkan n ke subbab berikutnya. Definisi Misalkan A dan B adalah adalah dua himpunan tak kosong. kosong. Mak Makaa hasil kali silang A B didefinisikan sebagai
×
A
× B = {(a, b)|a ∈ A dan b ∈ B }. Lebih umum lagi jika Ai untuk i = 1, 2, · · · , n adalah n buah himpunan tak kosong maka definisikan A1
× A2 × · · · × An = {(a1, a2, · · · , a2)|ai ∈ Ai} Contoh 1.2.1 Jika A = {1, 2, 3} dan B = {2, 3} maka A × B = {(1, (1, 2), 2), (1, (1, 3), 3), (2, (2, 2), 2), (2, (2, 3), 3), (3, (3, 2), 2), (3, (3, 3)} Definisi Misalkan Ai untuk i = 1, 2, · · · , n adalah n buah himpunan tak kosong, dan juga misalkan himpunan O = A1 × A2 ×···× An , jika u ∈ O maka u disebut n-tupel atau pasangan terurut, ditambah lagi u = (a1 , a2 ,
· · · , a2 )
ai
∈ Ai .
1.3. KETAKSAMAAN KETAKSAMAAN RATAAN RATAAN ARITMATIK ARITMATIK - RATAAN RATAAN GEOMETRIK 5 Definisi Misalkan u = (a ( a1 , a2 , , an ) dan v = (b1 , b2 , , bn ) adalah sebarang pasangan terurut, kita katakan u = v jika dan hanya jika ai = bi untuk setiap i = 1, 1 , 2, , n.
···
···
···
Definisi Misalkan u = (a ( a1 , a2 , , an ) dan v = (b1 , b2 , , bn ) adalah sebarang pasangan terurut, maka produk skalar dari u dan v didefinisikan sebagai
···
···
u v = a1 b1 + a2 b2 +
· · · + anbn
sedangkan norm- p norm- p dari u didefinisikan sebagai
· · ·
||u|| p p =
p
(u) (u)
(u)
p buah
jika p = 2 maka norm-2 dari u cukup ditulis u .
|| ||
Definisi Misalkan u = (a ( a1 , a2 , , an ) adalah sebarang pasangan terurut, permutasi dari u adalah semua pasangan terurut yang dapat diperoleh melalui penukaran posisi dari ai suatu dengan aj .
···
Contoh 1.2.2 Permutasi dari pasangan terurut (1, (1, 2, 3) adalah (1, (1, 3, 2)
(1, 2, 3) (2, 3, 1)
(2, 1, 3)
(3, 2, 1) (3, 1, 2)
Definisi Misalkan u = (a ( a1 , a2 , , an ) adalah sebarang pasangan terurut, permutasi siklis dari u adalah semua pasangan terurut yang bebentuk
···
(a1 , a2 ,
1.3 1.3
· · · , an)
(an , a1 ,
· · · , an−1) · · ·
(a3 , a4 ,
· · · , a2 )
(a2 , a3 ,
· · · , a1 ).
Keta Ketaks ksam amaa aan n Ra Rata taan an Arit Aritma mati tik k - Ra Rata taan an Geometrik
Sebagian besar orang yang mulai untuk belajar pertidaksamaan memulai dengan belajar ketaksam ketaksamaan aan Arithmatic Arithmatic Mean-Geometr Mean-Geometric ic Mean, Mean, atau disingkat disingkat AMGM. Banyak soal - soal susah yang dapat diselesaikan secara elegan dengan AM-GM. ···+an Rataan Aritmatik atau Arithmatic Mean didefinisikan sebagai a1 +a2 + n dan rataan geometri atau Geometric mean didefinisikan sebagai n a1 a2 an . Sedangkan ketaksamaan AM-GM dinyatakan didalam teorema berikut
√
···
Teorema 1.3.1 Jika a Jika a1 , a2 , , an adalah bilangan - bilangan riil positif, maka berlaku a1 + a2 + + an n a1 a2 an n
···
···
≥√
···
dengan kesamaan terjadi jika dan hanya jika a1 = a2 =
· · · = an
6
BAB 1. KETAKSAMA KETAKSAMAAN AN RAT RATAAN
Bukti Kita akan gunakan induksi matematika, untuk kasus n = 2, kita memperoleh bentuk a+b ab 2 yang benar karena setara dengan ( a b)2 0. Misalkan An+1 =
√ ≥ √ −√ ≥
a1 + a2 + + an+1 n+1
···
Gn =
√a1a2 · · · an+1
n+1
Dengan asumsi bahwa AM-GM benar untuk n buah variabel (hipotesis induksi) kita perole p eroleh h n 1 buah
−
· · · ≥ −
an+1 + (n (n 1)A 1)An+1 an+1 + An+1 + + An+1 n = an+1 (An+1 )n−1 n n an+1 + (n (n 1)A 1)An+1 Dengan memisalkan A = dan G = n an+1 (An+1 )n−1 maka n An + A An+1 = (An A)1/2 2 (Gn G)1/2 −1 )1/n )1/2 = (Gn (an+1 Ann+1
−
≥
≥
n−1 1/2n = (Gnn+1 +1 An+1 ) n−1 Yang berarti (A (An )2n Gnn+1 An+1 Gn+1 . +1 An+1 = Sekarang Sekarang akan ditunjukk ditunjukkan an bahwa bahwa kesamaa kesamaan n terjadi terjadi jika jika dan hanya hanya jika jika semua ai sama untuk untuk i = 1, 2, , n. Untu Untuk k n = 2, dapat dapat dilihat dilihat bahwa bahwa a = b ( a b)2 = 0. Jik Jika semua semua ai sama untuk i = 1, 2, ,n + 1 maka jelas bahwa An+1 = Gn+1 . Sekarang Sekarang misalkan misalkan An+1 = Gn+1 , maka dari rantai ketaksamaan diatas kita peroleh
≥
⇒
≥
···
⇐⇒ √ − √
An = A
···
An = Gn
A=G
Karena An = Gn maka berdasarkan hipotesis induksi a1 = a2 = dari A = G diperoleh an+1 = An+1 =
· · · = an. Lalu
a1 + a2 + + an + 1 nan + an+1 = n+1 n+1
···
jadi an = an+1 , melengkapi bukti kita. Berikutnya akan diberikan beberapa contoh untuk mengilustrasikan penggunaannya. Contoh Contoh 1.3.1 (Mandelbrot (Mandelbrot Problem Problem Corner, Thomas J. Mildorf) Misalkan a , b, dan c adalah bilangan riil positif, tentukan nilai minimum yang mungkin dari a3 b 1+c + 2+ 4b 8c 2a dan tentukan nilai a, b dan c pada saat nilai minimum tersebut tercapai.
1.3. KETAKSAMAAN KETAKSAMAAN RATAAN RATAAN ARITMATIK ARITMATIK - RATAAN RATAAN GEOMETRIK 7 Solusi Perhatikan bahwa dengan aplikasi AM-GM a3 b 1+c a3 b 1 c c + 2+ = + 2+ + + 4b 8c 2a 4b 8 c 2a 4a 4a
≥ 5
5
a3 b 1 c c 5 = 2 4b 8c 2a 4a 4a 4
· · · · · · · ·
Karena kesamaan terjadi jika dan hanya jika a3 b 1 c c = 2 = = = 4b 8c 2a 4a 4a
⇐⇒ a = 2, b = 8, c = 2.
Contoh 1.3.2 (International Math Olympiad 2001) Buktikan bahwa b c √a2 a+ 8bc ≥1 +√ +√ 8bc b2 + 8ac 8ac c2 + 8ab 8ab untuk a, b dan c bilangan riil positif. Solusi Kita akan buktikan bahwa a 2 a + 8bc 8bc
√
≥
a4/3 a4/3 + b4/3 + c4/3
Perhatikan bahwa dengan AM-GM kita peroleh (a4/3 + b4/3 + c4/3 )2
− (a4/3)2
= (b4/3 + c4/3 )(a )(a4/3 + a4/3 + b4/3 + c4/3 ) 2(bc 2(bc))2/3 4a2/3 (bc) bc)1/3 = 8a2/3 bc
≥
Jadi (a4/3 + b4/3 + c4/3 )2
4/3
a a ≥ (a4/3)2 + 8a ≥ 8a2/3 bc ⇐⇒ √ 4 / 3 2 a + b4/3 + c4/3 a + 8bc 8bc
Dengan Dengan cara yang sama diperoleh diperoleh 4/3
b √b2 +b 8ac ≥ a4/3 + b4/3 + c4/3 8ac
4/3
b √c2 +c 8ab ≥ a4/3 + b4/3 + c4/3 8ab
Jadi apabila semua hasil diatas ditambahkan diperoleh 4/3
b c a √a2 a+ 8bc ≥ +√ +√ + 4 3 / a + b4/3 + c4/3 8bc b2 + 8ac 8ac c2 + 8ab 8ab a4/3 a4/3 + a4/3 + b4/3 + c4/3 a4/3 + b4/3 + c4/3 =1 melengkapi solusi.
8
BAB 1. KETAKSAMA KETAKSAMAAN AN RAT RATAAN
Mungkin pada awalnya sulit bagi kita untuk mendapatkan ide awal dari solusi soal IMO 2001 diatas, angka 4/ 4 /3 tampak tidak alami untuk ditemukan. Solusi lain dari masalah ini yang tidak kalah elegan akan diberikan nanti. Untuk melihat bagaimana ide 4/ 4 /3 itu diperoleh, perhatikan bahwa pertama - tama kita mencoba mencari r sehingga r
a √a2 a+ 8bc ≥ ⇐ ⇒ (ar + br + cr )2 ≥ a2r + 8a 8a2r−2 bc r r r a +b +c 8bc Yang setara dengan (ar + br + cr )2
− a2r = (br + cr )(a )(ar + ar + br + cr ) ≥ 8a2r−2 bc
Tapi dengan AM-GM kita peroleh r/2 r/4 ≥ 8ar/2 (bc) bc)3r/4 Ini berarti nilai r yang mungkin harus memenuhi 2r = 2r − 2 dan 34r = 1 yaitu r = 4/3. Misalkan pada AM-GM kita mengganti a1 , a2 , · · · , an dengan a1 , a1 , · · · , a1
(br + cr )(a )(ar + ar + br + cr )
1
maka kita dapatkan
1 a1
+
1 a2
+ n
· · · + a1 ≥ n
n
1 1 a1 a2
2
n
· · · a1n
yang setara dengan
√a1a2 · · · an ≥ n
perhatikan bahwa bentuk
1
a1
1 a1
n + a1 + + a1n 2
···
+
1 a2
n +
· · · + a1
n
sering disebut rataan harmonik atau
Harmonic Mean (HM) , karena AM GM dan GM H M maka juga berlaku AM H M , kita nyatakan ketaksamaan GM -HM ini yang telah duperoleh diatas.
≥
≥
≥
Teorema 1.3.2 Jika a Jika a1 , a2 , berlaku n a1 a2
· · · , an adalah bilangan - bilangan riil positif, maka √ ···a ≥ n n 1 1 1 a + a +··· + a dengan kesamaan terjadi jika dan hanya jika a1 = a2 = · · · = an . Misalkan kita diminta untuk membuktikan A ≥ B , sedangkan dengan mengaplikasikan suatu teorema, yang kita dapatkan adalah A ≥ C , pembuktian selesai jika kita berhasil membuktikan C ≥ B . Nam Namun, un, jika ternyat ternyata a yang benar b enar adalah B ≤ C , apa yang harus kita lakukan? Kita dapat menggunakan teorema lain, atau tetap dengan teorema yang sama tapi diterapkan dengan cara yang berbeda untuk memperoleh A ≥ D dan D ≥ B . Jadi dalam mengerjak mengerjakan an soal 1
2
n
pertidaksamaan, kita tidak hanya bergantung pada alat - alat yang digunakan, tapi juga yang terpenting adalah bagaimana cara menerapkan teorema-teorema
1.3. KETAKSAMAAN KETAKSAMAAN RATAAN RATAAN ARITMATIK ARITMATIK - RATAAN RATAAN GEOMETRIK 9 tersebut untuk mendapatkan jalan yang benar menuju solusi, sebagai contoh untuk membuktikan membuktikan a b c + + a2 + 2 b2 + 2 c2 + 2
≤ 2a a+ 1 + 2b b+ 1 + 2c c+ 1
memang dapat dilakukan tanpa perlu menggunakan AM-GM 2 . Tapi jika kita akan menggunakan AM-GM untuk menyelesaikan soal ini, maka harus diperhatikan hatikan bagaimana cara menerapk menerapkan an AM-GM di soal ini. Misalkan Misalkan AM-GM diterapkan pada penyebutnya dengan cara berikut a2 + 2 2
≥
√
2a
⇐⇒ a2 a+ 2 ≤ 2√1 2
dan kemudian melakukan cara serupa pada variabel b dan c, maka kita akan memperoleh a2
a b c + 2 + 2 +2 b +2 c +2
≤ 2√1 2 + 2√1 2 + 2√1 2 = 2√3 2
Berikutnya kita harus membuktikan 3 a b c √ ≤ + + 2a + 1 2b + 1 2c + 1 2 2 tapi ternyata ternyata ketaksamaan ketaksamaan diatas salah, salah, karena untuk a = b = c = 1 kita dapatkan dapatkan 3 2 2 yang jelas salah. Jadi penerapan AM-GM dengan cara diatas tidak menyelesa menyelesaik ikan an soal ini, namun namun ini bukan bukan berarti berarti AM-G AM-GM M tidak dapat diterapkan pada soal ini. Kita masih dapat menggunakan AM-GM dengan cara berikut a a a2 + 2 = (a (a2 + 1) + 1 2a + 1 2 a +2 2a + 1
≤ √
≥
⇐⇒
≤
b2 + 2 = (b (b2 + 1) + 1
≥ 2b + 1 ⇐⇒ b2 b+ 2 ≤ 2b b+ 1
c2 + 2 = (c (c2 + 1) + 1
≥ 2c + 1 ⇐⇒ c2 c+ 2 ≤ 2c c+ 1
Yang apabila semua ditambahkan kita peroleh ketaksamaan pada soal. Sebenarnya pemilihan cara penggunaan AM-GM yang tepat seperti diatas didasark didasarkan an oleh sesuatu, yaitu kasus kesamaan. kesamaan. Perhatik Perhatikan an bahwa bahwa pada soal tersebut kesamaan terjadi jika a = b = c = 1. Jadi jika jika kita ingin melakukan melakukan AM-GM AM-G M kita harus melakuk melakukann annya ya sedemikian sedemikian sehingga sehingga kita dapatk dapatkan kasus kasus 2 a = 1 sebagai kasus kesamaan. Penerapan AM-GM seperti ( a +1)+1 2a + 1 menghasilkan a2 = 1 sebagai kasus kesamaan sedangkan penerapan AM-GM a2 + 2 2 2a menghasilkan a2 = 2 sebagai kasus samadengan. Hasil observasi pada kasus samadengan dapat menjadi langkah awal untuk menyelesaikan soal pertidaksamaan, bukan hanya pada soal ini.
≥
≥ √
2
perhatikan bahwa
a a
2 +2
≤
a
2a+1
⇐⇒ (a − 1)2 ≥ 0
10
BAB 1. KETAKSAMA KETAKSAMAAN AN RAT RATAAN
Contoh 1.3.3 (Fajar Yuliawan) Jika a, b, c,d > 0 dan abcd = 1, 1 , buktikan 1 1 1 1 + + + a+b+c+9 b+c+d+9 c+d+a+9 d+a+b+9
≤ 13
Solusi (Oleh Hendrata Dermawan) Dermawan) Dengan menggunakan AM-HM kita dapatkan (a + 3) + (b (b + 3) + (c ( c + 3) 3
3
≥
1 a+3
+
1 b+3
1 c+3
+
yang setara dengan 1 9
1 1 1 + + a+3 b+3 c+3
lakukan hal yang serupa dengan dapatkan
≥
1 a+b+c+9
1 1 b+c+d+9 , c+d+a+9
dan
1 d+c+b+9
. Jadi Jadi kit kitaa
1 1 1 1 + + + a+b+c+9 b+c+d+9 c+d+a+9 d+a+b+9 1 1 1 1 1 + + + 3 a+3 b+3 c+3 d+3
≤
Sehingga cukup bagi kita untuk membuktikan 1 1 1 1 + + + a+3 b+3 c+3 d+3
≤1
Dengan mengkalikan silang dan mengeskpansi kita peroleh 108 + 6S 6S 2 + 27S 27S 1
≤ abcd + 3S 3S 3 + 9S 9S 2 + 27S 27S 1 + 81 ⇐⇒ 2S 3 + 3S 3S 2 + 1 ≥ 27
dengan S 1 = a + b + c + d , S 2 = ab + ac + bc + cd + ad + bd dan S 3 = abc + bcd + cda + dab. dab. Perhatikan bahwa S 3 mempunyai 4 suku, S 2 mempunyai 6 suku, jadi 2S 2S 3 + 3S 2 +1 mempunyai total 27, dan dengan AM-GM kita peroleh 2S 3 + 3S 3S 2 + 1
27 27 (abcd) abcd)15 = 27
≥
Para pembaca dapat memeriksa bahwa pada penggunaan AM-HM dan AMGM pada contoh diatas, kasus samadengan yaitu a = b = c = d = 1 terawetkan. terawetkan. Hal ini setidakny setidaknyaa mem memberik berikan an petunjuk petunjuk bahwa bahwa kita berada pada jalur pemecahan masalah yang tidak salah. Contoh 1.3.4 (Vietnam 1998) Misalkan x1 , x2 , langan riil positif yang memenuhi 1 1 + + x1 + 1998 x2 + 1998 Buktikan
1 · · · + xn +11998 = 1998
√x1x2 · · · xn ≥ 1998 n−1 n
· · · , xn (n ≥ 2) adalah bi-
1.3. KETAKSAMAAN KETAKSAMAAN RATAAN RATAAN ARITMATIK ARITMATIK - RATAAN RATAAN GEOMETRIK 1111 1998 xi +1998
Solusi Misalkan yi = berdasarkan AM-GM
maka
n j =1 yj
= 1 dan 1
n
1 kalikan semua (1
− yi =
− yi =
j =i yj
jadi
n
yj
≥ (n − 1)
j =i
n−1
yj
j =i
− yi) untuk i = 1,1 , 2, · · · , n diperoleh
n
− 1
n
yi
i=1
n
≥ (n − 1)
n−1
n
yin 1
i=1
− = (n ( n − 1)n
yi
i=1
dengan mensubtitusi balik yi diperoleh (x1 x2 xn ) (x1 + 1998)(x 1998)(x2 + 1998) (xn + 1998)
···
···
yang setara dengan
(n − 1)n(1998)n ≥ (x1 + 1998)(x 1998)(x2 + 1998) · · · (xn + 1998)
√x1x2 · · · xn ≥ 1998 1998.. n−1 n
Contoh 1.3.5 (AMM Januari 2006, Marian Tetiva) Misalkan a,b,c Misalkan a,b,c adalah panjang dari sisi-sisi segitiga dengan R adalah jari-jari lingakran luar dan r adalah jari-jari lingkaran dalam. Buktikan R r
≥
4a3
4a2 (b
4b 2 (a
2
4c 2 (a
− − c) 2 · 4b 3 − − c) 2 · 4c 3 − − b ) 2
Solusi Oleh Fajar Yuliawan uliawan Misalkan s menyatakan setengah keliling dari segitiga dan A adalah luas-nya. A Maka berlaku rumus R = abc 4A dan r = s , jadi R abc s = = 2r 8A2 8s(s
·
−
abc s )(s b)(s )(s a)(s
· −
− c)
=
8(s 8(s
abc )(s b)(s )(s a)(s
− c) s(s − a)(s )(s − b)(s )(s − c). Ke−
−
dimana kita telah menggunakan rumus Hero A = mudian kita klaim bahwa ketaksamaan berikut berlaku a b)(s )(s
≥ − −
4a2 (b
2
− c) 2 . Untuk membuktikan ini, misalkan x = 2 (s b)(s )(s − c), maka x = (2s (2s − 2b)(2s )(2s − 2c) = (a + c − b)(a )(a + b − c) = a2 − (b − c)2
− (s
≥
− c)
4a3
dan ketaksamaan yang akan kita buktikan adalah a x
4a2 3a2 + x2
⇔
(3a (3a2 + x2 )2
√ ≥ 16 16a a3 x ⇔ 3a2 + x2 ≥ 4 a3 x
12
BAB 1. KETAKSAMA KETAKSAMAAN AN RAT RATAAN
Sedangkan dengan AM-GM kita peroleh 3a2 + x2 = a2 + a2 + a2 + x2
≥4
√ 4
√
a6 x2 = 4 a3 x.
Dengan cara yang serupa kita peroleh
(s
b a)(s )(s
(s
c b)(s )(s
− −
dan
− c) − a)
≥ ≥
4b3
4c3
2
4b2 (a
− − c)2 4c2 (b a)2
− −
2
kalikan ketiganya kita dapatkan ketaksamaan pada soal.
1.4
Ketaks Ketaksam amaan aan Akar Akar Rataan Rataan -AM
Ketaksamaan lain yang akan dibahas adalah sebuah ketaksamaan yang berbentuk mirip dengan AM. Ketaksamaan ini berhubungan dengan Root Mean Square
a2 +a2 + +a2
···
n 1 2 atau disingka disingkatt RMS yang didefinisikan didefinisikan sebagai sebagai . Ka Kam mi akan akan n menyatakan ketaksamaan RMS-AM dan memberikan buktinya sebagai latihan
Teorema 1.4.1 Jika a Jika a1 , a2 ,
· · · , an adalah bilangan - bilangan riil , maka berlaku a21 + a22 + · · · + a2n ≥ |a1| + |a2| + · · · + |an|
n
n
dengan kesamaan terjadi jika dan hanya jika a1 = a2 =
· · · = an.
Berikut ini diberikan beberapa contoh, yang pertama diambil dari olimpiade nasional Inggris dan contoh kedua yang lebih susah diambil dari olimpiade nasional China. Contoh 1.4.1 (British Mathematical Olympiad 2003) Jika x, y, dan z adalah bilangan riil tak negatif yang memenuhi x2 + y2 + z 2 = 1, 1 , buktikan x2 yz + xy 2 z + xyz 2
≤ 13
Solusi Dengan Dengan RMS-AM RMS-AM kita peroleh 3(x 3(x2 + y 2 + z 2 )
≥ (x + y + z )2 .
Dengan x2 + y 2 + z 2 = 1 diperoleh x+y+z
≤
√
3
Kemudian dengan RMS-GM diproleh
x2 + y 2 + z 2 3
≥ √xyz ⇐⇒ xyz ≤ 3√1 3 . 3
13
1.4. KETAKSAMA KETAKSAMAAN AN AKAR RAT RATAAN -AM Jadi x2 yz + xy2 z + xyz 2 = xyz( xyz (x + y + z )
≤ 3√1 3
√
1 3
3=
Contoh 1.4.2 (China Team Selection Test 1998) Misalkan n 2 adalah bilangan bulat positif. Jika x1 , x2 , , xn adalah bilangan riil sehingga
≥
···
n 1
n
−
2
xi +
i=1
i=1
xi xi−1 = 1. 1.
Diberikan sebuah bilangan bulat k dengan 1 dari xk .
| |
≤ k ≤ n, tentukan nilai maximum
Solusi Perhatikan Perhatikan bahwa x1 2 + (x (x1 + x2 )2 + (x (x2 + x3 )2 +
· · · + (x (xn−1 + xn )2 + xn 2 = 2
Dengan menggunakan RMS-AM dan ketaksamaan segitiga kita peroleh
x1 2 + (x (x1 + x2 )2 + k x1 + x1 + x2 + k x1 (x1 + x2 ) +
· · · + (x (xk−1 + xk )2 | · · · + |xk−1 + xk |
≥| | | ≥| − |xk | =
· · · + (−1)k−1(xk−1 + xk )| k
k
Jadi kita peroleh x1 2 + (x (x1 + x2 )2 +
xk · · · + (x (xk−1 + xk )2 ≥ k
2
Dengan cara yang serupa kita peroleh 2
2
xn + (x (xn−1 + xn−2 )
xk + · · · + (x (xk+ + xk )2 ≥ k
2
.
Tambahkan kedua buah hasil tersebut diperoleh 2 = x1 2 + (x (x1 + x2 )2 + (x (x2 + x3 )2 +
≥
1 + k n
−
1 k+1
· · · + (x (xn−1 + xn )2 + xn 2
xk 2
jadi
|xk | ≤ 2k(nn−−k1+ 1)
k+1) Ini memberikan kita motivasi bahwa nilai maksimum dari xk adalah 2k(nn− −1 , untuk untuk mem membukti buktika kan n pernyataan pernyataan ini kita cukup memperlihatka memperlihatkan n bahwa bahwa nilai
| |
14
BAB 1. KETAKSAMA KETAKSAMAAN AN RAT RATAAN
tersebut memang dapat dicapai oleh xk yaitu dengan memperlihatkan memperlihatkan kasus kesamaa kesamaan. n. Perhatik Perhatikan an bahwa bahwa pada penggunaan penggunaan RMS-AM dan ketaksam ketaksamaan aan segitiga kita dapatkan bahwa kesamaan terjadi jika dan hanya jika x1 =
−(x1 + x2) = x1 + x3 = · · · = ( −1)k−1)(x )(xk−1 + xk )
dan xk + xk+1 =
−(xk+1 + xk+2) = · · · = ( −1)n−1xn
Oleh karena itu kesamaan terjadi jika dan hanya jika xi =
−
( 1)k−i xkk i n+1−i) ( 1)i−k xkn(− k+1
−
jika i = 1, 1 , 2, , k 1, jika i = k + 1, 1, , n.
··· − ···
Latihan 2 1. Tunjukkan bahwa untuk bilangan positif a, b, dan c berlaku (a + b + c)(ab )(ab + ac + bc) bc)
≥ 9abc
2. Jika a , b , c adalah bilangan riil positif yang memenuhi abc = 1
≤ a2 + b2 + c2
a+b+c
3. Jika a , b , c adalah adalah bilangan bilangan riil positif. Buktikan Buktikan (a + b + c)2
≥ 3(ab 3(ab + ac + bc) bc)
4. Buktikan ketaksamaan RMS-AM ! 5. Jika x,y, z > 0 buktikan x+
x + (x + y)(x )(x + z ) y +
y + (y + z )(y )(y + x) z +
z (z + y)(z )(z + x)
≤1
Hint: Gunakan cara yang mirip dengan pada soal IMO 2001, disini r = 12 . 6. Jika a1 , a2 , , an adalah bilangan riil positif buktikan
···
(a1 + a2 +
· · · + an)
1 1 + + a1 a2
···
1 + an
≥
n2
7. (Rusia) (Rusia) Jika x,y,z adalah bilangan riil positif yang memenuhi xyz = 1, buktikan (x + y )(x )(x + z )(y )(y + z ) 4(x 4(x + y + z 1) 8. 0,
≥ − (USAMO (USAMO 1998) 1998) Misalkan a0 , a1 , · · · , an adalah bilangan pada interval π
2
sehingga
− −
tan a0
π + tan a1 4
π + 4
· · · + tan
− an
π 4
15
1.5. KETAKSAMA KETAKSAMAAN AN JENSEN JENSEN
buktikan tan a0 tan a1 tan an nn−1 . Hint: Langkah yang yang digunakan mirip dengan dengan solusi Soal Vietnam Vietnam 1998. 9. (APMO 1998) Misalkan a,b,c adalah adalah bilangan bilangan riil positif. Buktika Buktikan n
···
≥
≥ a 1+ b
b 1+ c
c 1+ a
a+b+c 2 1+ 3 abc 10.(Mircea 10.(Mircea Lascu) Lascu) Jika a,b,c adalah bilangan riil positif dengan abc = 1, buktikan 1 1 1 1 + 2 + 2 . 2 2 2 2 a + 2b 2b + 3 b + 2c 2c + 3 c + 2a 2a + 3 2
√
≤
1.5 1.5
Keta Ketaks ksam amaa aan n Jens Jensen en
Ketaksamaan yang kita kenal sebelum ini masih dapat digeneralisasi lagi, ketaksamaan AM-GM merupakan alat yang ampuh untuk menyelesaikan soal soal pertidaksamaan yang terpisah, tapi ketaksamaan yang akan diperkenalkan berikut dapat membuat solusi -solusi yang lebih kreatif dan hasil - hasil yang lebih umum, sebelumnya kita mulai dengan memberikan sebuah definisi. Definisi Sebuah fungsi bernilai riil f dikatakan dikatakan konveks konveks di suatu interval interval I jika dan hanya jika untuk setiap x1 , x2 I dan 0 λ 1 berlaku λf ( λf (x1 ) + (1
∈
≤ ≤
− λ)f ( f (x2 ) ≥ f ( f (λx1 + (1 − λ)x2 )
Sedangkan fungsi f dikatakan konkav jika dan hanya jika λf ( λf (x1 ) + (1
− λ)f ( f (x2 ) ≤ f ( f (λx1 + (1 − λ)x2 )
Secara visual, grafik fungsi konveks merupakan sebuah kurva yang cekung keatas, atau jika dihubungkan dengan definisi, pada fungsi konveks apabila diambil sebarang dua titik pada kurva f maka garis yang menghubungkan dua titik tersebut pasti berada di atas kurva f itu sendiri. Teorema 1.5.1 (Ketaksamaan Jensen) Misalkan f adalah fungsi riil yang konveks pada interval I , jika x1 , x2 , , xn I dan λi dengan i = 1, 2, ,n adalah bilangan riil nonegatif yang memenuhi λ1 + λ2 + + λn = 1 maka berlaku
···
λ1 f (x1 ) + λ2 f ( f (x2 ) +
∈
···
···
· · · + λnf (xn) ≥ f ( f (λ1 x1 + λ2 x2 + · · · + λn xn )
Bukti Untuk kasus n = 2 kita memperoleh definisi, misalkan teorema benar sampai pada n buah variabel, jika λ1 + λ2 + + λn+1 = 1 maka n+1
i=1
· · · ≥ ≥ n
xi f ( f (xi )
λi f
i=1
n
f
λi
i=1 n+1
= f
xi λi .
i=1
n i=1 λi xi n i=1 λi
+ λn+1 f ( f (xn+1 )
n i=1 λi xi n i=1 λi
Q.E.D
+ λn+1 xn+1
16
BAB 1. KETAKSAMA KETAKSAMAAN AN RAT RATAAN
Pada teorema diatas tidak dinyatakan kapan kesamaan dapat tercapai, tapi dapat dilihat dengan mudah bahwa apabila x1 = x2 = = xn maka kesamaan akan tercapai. Sedangkan apabila fungsi tersebut memenuhi kriteria pada definisi fungsi konveks dengan tanda diganti < maka kesamaan hanya akan terjadi jika x1 = x2 = = xn . Untuk fungsi konkav ketaksamaan Jensen tetap berlaku dengan tanda diganti . Akan tetapi sulit bagi kita untuk menentukan apakah sebuah fungsi konvek dengan menggunakan definsi secara langsung, teorema-teorema berikut kamii nyatakan kam nyatakan tanpa bukti karena pembuktiannya pembuktiannya membutuhkan membutuhkan kalkulus yang terlalu jauh dari bahasan kita.
···
≤
···
≥
≤
Definisi Fungsi riil f dikatakan menaik di interval I jika untuk setiap x, y berlaku x
∈ I
⇒
dan dikatakan menurun jika berlaku x
⇒ f ( f (x) > f ( f (y )
Teorema 1.5.2 Jika f adalah fungsi riil dan f (x) > 0 untuk setiap x di I maka f adalah menaik di I , sebaliknya jika f (x) < 0 di I maka f menurun. Teorema 1.5.3 Fungsi riil f konveks di I jika dan hanya jika f (x) sedangkan f konkav di I jika dan hanya jika f (x) 0 di I .
≤
≥ 0 di I ,
Teorema 1.5.4 Jika f adalah fungsi menaik pada garis bilangan riil maka: jika f dan g konvek di interval I , maka f g juga konveks. jika f dan g konkav di interval I , maka f g juga konkav.
◦ ◦
Contoh 1.5.1 (USAMO 1974) Untuk a,b,c > 0 buktikan bahwa aa bb cc (abc) abc)(a+b+c)/3 !
≥
1 Solusi Ambil f ( f (x) = ln x, karena f (x) = − 0 untuk x > 0, maka fungsi f x2 konkav pada interval x > 0. Jadi berdasarkan ketaksamaan Jensen
≤
a ln a b ln b c ln c + + a+b+c a+b+c a+b+c
≥ ln
a2 + b2 + c2 a+b+c
Dari RMS-AM diperoleh a2 + b2 + c2 a+b+c Jadi
≥
a+b+c = 3
⇒ ln
a2 + b2 + c2 a+b+c
a ln a b ln b c ln c + + a+b+c a+b+c a+b+c
≥ ln
Dengan AM-GM kita peroleh a+b+c 3
≥
√ 3
abc =
⇒ ln
a+b+c 3
≥ ln
a+b+c 3
a+b+c 3
≥ √ ln
3
abc
17
1.5. KETAKSAMA KETAKSAMAAN AN JENSEN JENSEN Jadi
a b c ln a + ln b + ln c a+b+c a+b+c a+b+c Dengan penyederhanaan akan diperoleh aa bb cc
≥ ln
√ 3
abc
≥ (abc) abc)(a+b+c)/3
Sering kali soal - soal yang dapat diselesaikan dengan ketaksamaan Jensen dapat digeneralisasi menjadi banyak vaiabel, Contoh 1.5.2 India 1995 Misalkan x Misalkan x1 , x2 , , xn adalah bilangan riil positif dengan hasil penjumlahannya sama dengan 1. Buktikan bahwa
···
x1 + 1 x1
√ −
xn 1 xn
···+ √ −
≥
n
n
−1
Solusi Perhatikan Perhatikan bahwa fungsi f ( adalah fungsi konvek konvekss untuk untuk f (x) = √ 11−x adalah x < 1 karena f (x) = 4(1−3x)5/2 0 untuk x < 1, dimana kita tahu bahwa x1 + x2 + xn = 1 dan xi < 1 untuk i = 1, 1 , 2, , n maka berdasarkan ketaksamaan ketaksamaan Jensen x1 xn 1 + + 1 x1 1 xn 1 (x21 + x22 + + x2n )
≥
···
√ −
···
··· √ −
≥
Sedangkan dari RMS-AM kita peroleh (x21 + x22 +
−
···
· · · + x2n) ≥ (x1 + x2 +n· · · + xn)
· · · xn = 1 maka 1 ⇐⇒ (x21 + x22 + · · · + x2n ) ≥ n ⇐⇒ ⇐⇒
2
Karena x1 + x2 +
n(x21 + x22 +
· · · + x2n) ≥ 1 n(x21 + x22 + · · · + x2n ) − n ≥ 1 − n n 1 ≥ 2 2 1−n (x + x + · · · + x2 ) − 1 1
⇐⇒
n
⇐⇒
1
− 1 ≤ 1 − (x21 + x22 + · · · + x2n) n 1 ≤ 2 n−1 1 − (x + x2 + · · · + x2 )
Dan kemudian dapat disimpulkan
√1x−1 x
1
+
· · · + √1x−n x ≥ n
n
2
n
1
2
1
− ≥ 1
(x21
n
n
−1
+ x22 +
· · · + x2n)
n
18
BAB 1. KETAKSAMA KETAKSAMAAN AN RAT RATAAN
Pada subbab sebelumnya, telah diberikan sebuah bukti dari soal IMO 2001, seringkali kita tertarik untuk melihat pendekatan atau solusi lain dari suatu masalah. masalah. Solusi lain tersebut tersebut dapat memperkay memperkayaa kreativita kreativitass berpikir kita dan seringkali berguna apabila kita mengerjakan persoalan lain yang boleh jadi mempunyai ide yang serupa. Sangat dianjurkan bagi para pembaca untuk mencoba menemuk menemukan an solusi alternatif dari suatu soal. Berikut Berikut ini salah satu solusi lain dari soal IMO 2001 dengan menggunakan ketaksamaan Jensen. Contoh Contoh 1.5.3 (Generalisa (Generalisasi si soal International International Math Olympiad Olympiad 2001) Buktikan bahwa
√a2 a+ λbc + √b2 +b λac + √c2 +c λab ≥ √13+ λ untuk a, b dan c bilangan riil positif dan bilangan riil λ
≥ 8.
1 Solusi Fungsi f ( f (x) = √ konveks untuk x > 0 karena f (x) = x x > 0, aplikasikan ketaksamaan Jensen pada fungsi ini dengan
λ1 =
a a+b+c
λ2 =
x1 = a2 + λbc
b a+b+c
x2 = b2 + λac
λ1 =
3 4x5/2
> 0 untuk
c a+b+c
x3 = c2 + λab
maka akan didapatkan ketaksamaan
√a2 a+ λbc + √b2 +b λac + √c2 +c λab ≥ Jadi sekarang tinggal memeriksa apakah
a+b+c
a3 +b3 +c3 +3λabc +3λabc a+b+c
a+b+c
3 √ ≥ a +b +c +3λabc +3λabc 1+λ
3
3
3
a+b+c
Tapi dengan AM-GM kita peroleh ( a + b + c) 3
= (a3 + b3 + c3 ) + 3(ab 3(ab2 + ac2 + ba2 + bc2 + ca2 + cb2 ) + 6abc 6 abc (a3 + b3 + c3 ) + 18abc 18abc + 6abc 6abc 3 3 3 = (a + b + c ) + 24abc 24abc
≥
Dengan AM-GM kita peroleh a3 + b3 + c3 (λ + 1)(a 1)(a + b + c)3
≥ 3abc dan karena λ − 8 ≥ 0 maka
= (λ + 1)(a 1)(a3 + b3 + c3 ) + 24(λ 24(λ + 1)abc 1)abc 3 3 3 3 = 9(a 9(a + b + c ) + (λ ( λ 8)(a 8)(a + b3 + c3 ) + 24(λ 24(λ + 1)abc 1)abc 9(a 9(a3 + b3 + c3 ) + 3(λ 3(λ 8)abc 8)abc + 24(λ 24(λ + 1)abc 1)abc 3 3 3 = 9(a 9(a + b + c ) + 27λabc 27λabc = 9(a 9(a3 + b3 + c3 + 3λabc 3λabc)).
≥
− −
19
1.5. KETAKSAMA KETAKSAMAAN AN JENSEN JENSEN Sekarang kita mempunyai (λ + 1)(a 1)(a + b + c)3
2
( a + b + c) ≥ 9(a ≥ (λ +9 1) 9(a3 + b3 + c3 + 3λabc 3λabc)) ⇐⇒ a +b +c +3λabc +3λabc 3
3
3
a+b+c
Akarkan kedua ruas maka a+b+c
3 √ ≥ a +b +c +3λabc +3λabc λ+1
3
seperti yang diinginkan.
3
3
a+b+c
Contoh 1.5.4 (AM-GM terboboti) Buktikan bahwa untuk bilangan positif a1 , a2 , , an dan bilangan positif λ1 + λ2 + + λn = 1 berlaku
···
··· λ1 a1 + λ2 a2 + · · · + λn an ≥ aλ1 aλ2 · · · aλn dengan kesamaan terjadi jika dan hanya jika a1 = a2 = · · · = an . Solusi Pertimbangkan Pertimbangkan fungsi f ( f (x) = l n x, karena karena f (x) = − x1 1
n
2
< 0 untuk x > 0 maka fungsi tersebut konkav dan berdasarkan ketaksamaan Jensen, 2
λ1 ln x1 + λ2 ln x2 +
· · · + λn ln xn ≤ ln(λ ln(λ1 x1 + λ2 x2 + · · · + λn xn )
dimana λ1 , λ2 , adalah bilangan bilangan riil positif dengan dengan hasil penjumlahan penjumlahan , λn adalah sama dengan satu. Jadi kita peroleh
···
ln(x ln(xλ1 1 xλ2 2
· · · xλn ) ≤ ln(λ ln(λ1 x1 + λ2 x2 + · · · + λn xn ) n
sehingga dapat disimpulkan xλ1 1 xλ2 2
· · · xλn ≤ λ1x1 + λ2x2 + · · · + λnxn n
Untuk melihat kapan kesamaan terjadi, perhatikan bahwa fungsi f memenuhi f (x) > 0 jadi kesamaan hanya terjadi jika x1 = x2 = = xn .
···
Hasil diatas adalah generalisasi dari AM-GM, perhatikan bahwa jika kita memilih λ1 = λ2 = = λn = n1 kita kita peroleh peroleh AM-GM AM-GM yang yang biasa. biasa. Denga Denga Ketaksama Ketaksamaan an Jensen Jensen juga dapat diperoleh diperoleh RMS-AM RMS-AM dan GM-HM terboboti. terboboti.
···
Contoh 1.5.5 (International Mathematics Olympiad 2006) Tentukan bilangan riil M terkecil sehingga ketaksamaan
|ab( ab(a2 − b2 ) + bc( bc(b2 − c2 ) + ca( ca(c2 − a2 )| ≤ M (a2 + b2 + c2 )2 berlaku untuk sembarang bilangan riil a , b dan c
20
BAB 1. KETAKSAMA KETAKSAMAAN AN RAT RATAAN
Solusi Solusi resmi Pertama - tama kita akan tunjukkan bahwa
|ab( ab(a2 − b2 ) + bc( bc(b2 − c2 ) + ca( ca(c2 − a2 )| = |(a − b)(b )(b − c)(a )(a − c)(a )(a + b + c)| Pertimbangkan P ( P (t) = bt( bt(t2
− b2) + bc( bc(b2 − c2 ) + ct( ct(c2 − t2 )
Perhatikan P ( P (t) adalah polinomial derajat 3 dalam t dan juga P ( P (b) = P ( P (c) = P ( P ( b c) = 0 dab juga P ( P (t) = 0 jika b = c. Jadi
−−
P ( P (t) = (b
− c)(t )(t − b)(t )(t − c)(t )(t + b + c)
sehingga diperloleh
|ab( ab(a2 − b2 ) + bc( bc(b2 − c2 ) + ca( ca(c2 − a2 )| = |P ( P (a)| = |(b − c)(a )(a − b)(a )(a − c)(a )(a + b + c)| Sekarang masalahnya ekuivalen dengan mencari M terkecil sehingga
|(b − c)(a )(a − b)(a )(a − c)(a )(a + b + c)| ≤ M (a2 + b2 + c2 )2 Cukup bagi kita untuk hanya mempertimbangkan kasus dimana a karena untuk kasus lain contohnya a c b kita dapat menuliskan
≥ b ≥ c,
≥ ≥ |(b − c)(a )(a − b)(a )(a − c)(a )(a + b + c)| = |(c − b)(a )(a − b)(a )(a − c)(a )(a + b + c)| yang berarti tidak masalah jika posisi b digantikan oleh c, dan jika b ≥ a ≥ c
kita dapat menuliskan
|(a − c)(b )(b − a)(b )(b − c)(a )(a + b + c)| = |(b − c)(a )(a − b)(a )(a − c)(a )(a + b + c)| Kasus - kasus lain ditangani serupa, dimana posisi a , b dan c dapat saling dipertuk dipertukark arkan an tanpa mengubah persoalan persoalan awal. Jadi cukup bagi kita untuk untuk membuktikan kasus a b c. Oleh kerena itu (a (a b) 0 dan (b (b c) 0 lalu dengan AM-GM kita dapatkan
≥ ≥
|(a − b)(b )(b − c)| = (a − b)(b )(b − c)
− ≥
≤
(a
− b) + (b(b − c)
dengan kesamaan terjadi jika dan hanya jika a Kemudian dengan RMS-AM kita peroleh (a
− c)2 = 4
(a
− b) + (b(b − c) 2 ≤ 2
2
− ≥
2
(a
− c) 2 4
− b = b − c atau a + c = 2b. (a
− b)2 + (b(b − c)2 2
Jadi 3(a 3(a
=
− c)2 ≤ 2((a 2((a − b)2 + (b (b − c)2 + (a (a − c)2 )
21
1.5. KETAKSAMA KETAKSAMAAN AN JENSEN JENSEN
Dengan kesamaan juga terjadi jika dan hanya jika a + c = 2b 2 b. Dari hasil - hasil diatas kita dapatkan
|(b − c)(a )(a − b)(a )(a − c)(a )(a + b + c)| ≤ 14 |(a − c)3(a + b + c)| 1 = (a − c)6 (a + b + c)2 4 2 2 ( a − c) 2 ≤ 1 8 (a − b) + (b(b − c) + (a
√ 4
2 = 2
4
3
(a
−
b)2
+ (b (b
−
c)2
+ (a (a
3
−
3
(a + b + c)2
( a + b + c) 2
Dengan Dengan AM-GM terboboti kita peroleh
|(b − c)(a )(a − b)(a )(a − c)(a )(a + b + c)| √2 (a − b)2 + (b (b − c)2 + (a ( a − c) 2 ≤
√ − √ − ≤ 4
2
2 = 2 2 2
=
√ 9 2 32
(a
3 4
(a
3
b)2 + (b (b
− c)2 + (a ( a − c) 2 3
b)2 + (b (b
3
3/4
− c)2 + (a (a − c)2 3
2
3
c)2
(a + b + c)2
2
2
((a ((a + b + c)2 )1/4
1 + ( a + b + c) 2 4
2
(a2 + b2 + c2 )
√
Kita lihat bahwa M = 9322 memenuhi. Untuk melihat bahwa M tersebut adalah yang yang terkecil, terkecil, kita harus tunjukkan tunjukkan bahwa bahwa tanda tanda samadengan samadengan dapat terjadi, terjadi, yaitu terdapat a, b dan c sehingga
|(b − c)(a )(a − b)(a )(a −
√
9 2 2 c)(a )(a + b + c) = (a + b2 + c2 ) 32
|
Untuk menyelidiki ini, kita lihat beberapa ketakamaan yang telah kita gunakan untuk mendapatkan nilai M yaitu AM-GM dua kali dan AM-GM terboboti. Pada penggunaan 2 kali AM-GM kita dapatkan bahwa kesamaan terjadi jika dan hanya jika a+c = 2b, dan dari penggunaan AM-GM terboboti kita dapatkan 2 +(b−c)2 +(a +(a−c)2 bahwa kesamaan terjadi jika dan hanya jika (a−b) +(b = (a2 + b2 + 3 2 2 c ) , jadi kesamaan terjadi jika dan hanya jika a+c b= 2
dan
(a
− b)2 + (b(b − c)2 + (a ( a − c) 2 = (a ( a2 + b2 + c2 )2 3
22
BAB 1. KETAKSAMA KETAKSAMAAN AN RAT RATAAN
Subtitusikan b = diperoleh
a+c 2
ke persamaan persamaan 2(c 2(c
(a b)2 +(b +(b c)2 +(a +(a c)2 3
−
−
−
= (a2 + b2 + c2 )2
− a)2 = 9(a 9(a + c)2
Jadi kondisi kesamaan adalah b=
a+c 2
− a)2 = 18b 18 b2 √ 1 − 3 2 dan b = (c
√
Pada saat b = 1 kita peroleh a = 1 + 32 2 merupakan merupakan 2 salah satu nilai a, b dan c yang yang memen memenuhi uhi.. Nilai Nilai b yang lain dapat menghasilkan nilai a dan c yang yang berbeda pula. pula. Jadi Jadi kesam kesamaan aan terjadi terjadi pada nilai 3 3 (a,b,c) a,b,c) = 1 2 2, 1, 1 + 2 2 dan pasangan terurut yang proposional. Juga perhatikan bahwa kita telah mengasumsikan a b c karena perubahan posisi a, b dan c dapat dipertukarkan, jadi (a,b,c (a,b,c)) = 1 32 2, 1 + 32 2, 1 juga dapat menyebabkan menyebabkan kesamaan. Sehingga kita simpulkan simpulkan kesamaan terjadi jika dan hanya jika nilai a,b,c adalah proposional dengan 1 32 2, 1, 1 + 32 2 dan permutasinya.
−√
√
≥ ≥
−√ −√
√
√
Latihan 3 1. Buktikan ketaksamaan Power Mean - Arithmatic Mean yaitu untuk bilangan riil positif a1 , a2 , , an dan p > 1 atau p < 0 berlaku
···
a p1 + a p2 + n
· · · + a pn ≥
a1 + a2 + n
· · · + an
p
dengan kesamaan terjadi jika dan hanya jika a1 = a2 = = an . 2. Jika α, β dan γ adalah sudut - sudut dari segitiga lancip. Buktikan
···
cos α + cos β + cos γ
≤ 32 .
3. Misalkan pada sebuah lingkaran diambil n buah titik, kemudian dari tiap tiap titik tersebut ditarik garis ke pusat lingkaran sehingga diperoleh poligon didalam lingkaran tersebut. Buktikan bahwa poligon dengan luas terbesar diperoleh jika semua titik yang bersebelahan mempunyai jarak sama (atau dengan kata lain apabila apabila poligon yang dimaksud adalah p oligon teratur). teratur). 4. (Korea 1998). 1998). Jika Jika a,b,c adalah bilangan riil memenuhi a + b + c = abc maka buktikan 1 1 1 3 + + 2 1 + a2 1 + b2 1 + c2 Dan tentukan kapan terjadi kesamaan. √ √ 1 2 2 Peringatan: Fungsi f ( f (x) = √ 1+x tidak konveks pada < x < Lihatt 2 2 . Liha 1+x2 soal 2. 5. Jika x,y,z adalah bilangan riil positif dengan kondisi x + y + z = 1 , buktikan
√
√
√
≤
−
x + 1 x
√ −
y + 1 y
√ −
z 1 z
√ −
≥ 3 2
23
1.5. KETAKSAMA KETAKSAMAAN AN JENSEN JENSEN
6.( 6.(Germany Germany 2005) 2005) Misalkan a,b,c adalah bilangan riil positif dengan ab + ac + bc = 1, buktikan
3
1 + 6b 6b + a
3
1 + 6c 6c + b
3
1 + 6a 6a c
1 ≤ abc
Dan tentukan kapan kasus samadengan terjadi. 7. Untuk x y 1, buktikan
≥ ≥
√xx+ y + √yy+ 1 + √x1+ 1 ≥ √xy+ y + √xx+ 1 + √y1+ 1 . Hint: Misalkan x = y + a dan y = 1 + b dengan a, b
≥ 0.
24
BAB 1. KETAKSAMA KETAKSAMAAN AN RAT RATAAN
Bab 2
Ketaksamaan Norm 2.1
Ketaksamaan Cauchy-Sch Cauchy-Schwarz warz (CS), Minkowski Minkowski dan H¨ older older
Teorema 2.1.1 Jika diberikan sebuah pasangan terurut u = (a1 , a2 , dan v = (b1 , b2 , , bn ) adalah bilangan riil, maka berlaku
···
uv
· · · , an)
≤ |u||v|
atau ditulis
≤ 2
n
n
ak bk
n
ak
k=1
k=1
bk
k=1
dengan dengan kesamaan kesamaan terjadi terjadi jika ai = c bi untuk suatu konstan c. n
Bukti Kita mulai dari
(ak x + bk )2
k=0
≥ 0, oleh karena karena itu kita dapatk dapatkan
n
(ak2 x2 + 2a 2ak bk x + bk2 )
k=0
we define n
A=
k=0
n
ak2
B=
n
bk2
k=0
C =
ak bk
k=0
n
kemudian
(ak x + bk )2 = Ax2 + 2Bx 2 Bx + C
k=0
≥ 0, jelas bahwa A ≥ 0, .
Jik Jika
A = 0 , maka semua ai sama dengan 0 jadi C = 0 dan B = 0 sehingga tidak 25
26
BAB 2. 2. KETAKSAM KETAKSAMAAN AAN NORM NORM
ada yang perlu dibuktikan. Sekarang misalkan A > 0 maka 2
− − −
Ax + 2Bx 2Bx + C = A x2 + 2
B C x+ A A
B = A x +2 x+ A 2
= A
B x+ A
2
+
2
B A
B A
2
+
C A
AC B 2 A2
AC B 2 A 0+ A2 AC B 2 = A
≥
−
2
−B adalah nilai minimum dari Ax2 + 2Bx + C yang tercapai pada saat Jadi AC A x = −AB . Karena kita ketahui bahwa Ax2 + 2Bx 2Bx + C 0 untuk setiap x, maka 2 AC B pada saat x = −AB kita peroleh 0 atau AC B 2 . Subtitusi Subtitusi balik balik A A, B , dan C didapatkan
≥
−
≥
≥
≤ 2
n
n
ak bk
k=1
n
ak2
k=0
bk2
k=0
seperti yang diinginkan. Perlu ditekankan lagi bahwa ketaksamaan CS berlaku untuk semua bilangan riil, ditambah ditambah lagi kita mempunyai mempunyai dua buah variabel variabel yang yang memiliki memiliki indeks yaitu ai dan bi , kepiawaian kita dalam menentukan ai dan bi akan membuat ketaksamaan ini menjadi sangat berguna. Contoh 2.1.1 Jika a,b,c adalah bilangan - bilangan riil maka buktikan abc( abc(a + 2 2 2 b + c) (ab) ab) + (ac (ac)) + (bc (bc))
≤
Solusi Dengan menggunakan ketaksamaan Cauchy-Schwarz kita dapatkan xy+ xy + yz+ yz +zx x2 + y 2 + z 2 y 2 + z 2 + x2 = x2 +y 2 +z 2 . Jadi kita dapatkan untuk sebarang bilangan riil x,y,z
≤
xy + yz + xz
≤ x2 + y2 + z2.
Karena x,y,z adalah bilangan riil sebarang, maka kita boleh memisalkan memisalkan x = ab, ab, y = ac dan z = bc. bc. Jadi abc( abc(a + b + c) = (ab)( ab)(ac ac)) + (ac (ac)( )(bc bc)) + (ab ( ab)( )(bc bc))
≤ (ab) ab)2 + (ac (ac))2 + (bc (bc))2 .
¨ 2.1.. KETAKS 2.1 KETAKSAMA AMAAN AN CAUCH CAUCHY-S Y-SCHW CHWARZ ARZ (CS), (CS), MIN MINKO KOWSK WSKII DAN DAN H OLDER 27 Misalkan pada ketaksamaan Cauchy- Schwarz kita menggunakan pasangan a1 √ an terurut ( b1 , b2 , , bn ) dan ( √ , ab2 , , √ ) dimana ak adalah bilanb1 bn 2 gan riil dan bk > 0 maka diperoleh
√ √ ··· √
···
·√ ≤ n
bk
k=1
2
ak bk
n
n
bk
k=1
k=1
ak2 bk
Yang ekuivalen dengan a21 a22 + + b1 b2
2 · · · + abnn ≥ ab11 ++ ab22 ++ ·· ·· ·· ++ bann . √ a ⇐⇒ b = c a , Dengan Dengan kesamaan kesamaan terjadi terjadi jika jika b = c √
untuk untuk suatu konstanta c. Perha Perhatik tikan an baik baik - baik baik hasil hasil yang yang baru baru kita kita dapatk dapatkan an tersbu tersbut, t, ketaksaman tersebut akan sering berguna, terutama untuk mereduksi bentuk pecahan terpisah menjadi pecahan tunggal. Biasanya bentuk ini disebut ketaksamaan samaan CS bentuk bentuk Engel. Kita ilustrasikan ilustrasikan penggu p enggunaann naannya ya pada pembuktia pembuktian n salah satu ketaksamaan populer berikut. k
k
k
bk
k
Contoh 2.1.2 (Ketaksamaan Nesbitt) Buktikan untuk bilangan riil positif a , b dan c, a b c 3 + + b+c a+c a+b 2 Dengan kesamaan terjadi jika dan hanya jika a = b = c.
≥
Bukti Perhatikan bahwa a b c + + b+c a+c a+b
=
≥
a2 b2 c2 + + ab + ac ab + bc ac + bc (a + b + c)2 (ab + ac) ac) + (ab ( ab + bc) bc) + (ac (ac + bc) bc)
Dimana kita telah mereduksi tiga bentuk pecahan menjadi satu bentuk pecahan. Jadi cukup bagi kita untuk membuktikan (a + b + c)2 (ab + ac) ac) + (ab ( ab + bc) bc) + (ac (ac + bc) bc)
≥ 32
yang ekuivalen dengan ( a + b + c) 2
≥ 3(ab 3(ab + ac + bc) bc)
Sedangkan dari ketaksamaan CS diperoleh ab + ac + bc
≤ a2 + b2 + c2 ⇐⇒ 3(ab 3(ab + ac + bc) bc) ≤ (a + b + c)2
Dengan kesamaan terjadi jika j ika a2 = k (ab+ ab+ac), ac), b2 = k(bc+ bc+ab) ab) dan c2 = k (ac+ ac+bc). bc). Yaitu jika a = k (b + c), b = k(b + c) dan c = k(b + c), kita simpulkan kesamaan terjadi untuk a = b = c.
28
BAB 2. 2. KETAKSAM KETAKSAMAAN AAN NORM NORM
Contoh 2.1.3 (IMO Short List 1990) Jika ab + bc + cd + ad = 1 dengan a, b, c,d > 0 maka buktikan a3 b3 c3 d3 + + + b+c+d c+d+a d+a+b a+b+c
≥ 13
Solusi Pertama - tama perhatikan bahwa a3 b3 c3 d3 + + + b+c+d c+d+a d+a+b a+b+c a4 b4 c4 d4 = + + + ab + ac + ad bc + bd + ba cd + ca + cb da + db + dc Dengan menggunakan CS-Engel diperoleh a4 b4 c4 d4 + + + ab + ac + ad bc + bd + ba cd + ca + cb da + db + dc (a2 + b2 + c2 + d2 )2 2(ab 2(ab + ac + cd + ad + bc + bd) bd)
≥
Melalui ketaksamaan AM-GM dan CS a2 + b2 + c2 + d2
≥ ≥
2 a2 + b2 c2 + d2 2(ac 2(ac + bd) bd).
Jadi sekarang kita dapatkan (a2 + b2 + c2 + d2 )2 (a2 + b2 + c2 + d2 )2 = 2(ab 2(ab + ac + cd + ad + bc + bd) bd) 2 + 2(ac 2(ac + bd) bd)
2
2
2
2 2
≥ 2(a+ a+2 b+ b+2 c+ c+2 d+ )d2 .
Sedangkan dengan CS diperoleh a2 + b2 + c2 + d2 ab + bc + cd + ad = 1. 2 2 2 2 Misalkan X = a + b + c + d , maka (X (X 1) 0, dan jelas bahwa 3X 3 X + + 2 0. 2 Jadi 3X 3X X 2 = (3X (3X + + 2)(X 2)(X 1) 0. Maka
− −
− ≥
3X 2
≥
− ≥
≥
2
1 − X − 2 ≥ 0 ⇐⇒ 2 X ≥ + X 3
Dengan kata lain kita telah menunjukkan a3 b3 c3 d3 + + + b+c+d c+d+a d+a+b a+b+c
≥ =
≥ Seperti yang diinginkan.
(a2 + b2 + c2 + d2 )2 2 + a2 + b2 + c2 + d2 X 2 2 + X 1 . 3
¨ 2.1.. KETAKS 2.1 KETAKSAMA AMAAN AN CAUCH CAUCHY-S Y-SCHW CHWARZ ARZ (CS), (CS), MIN MINKO KOWSK WSKII DAN DAN H OLDER 29 Kita telah melihat bahwa CS bentuk Engel telah membuat bebearapa soal menjadi mudah, bahkan beberapa diantaranya adalah soal - soal dari IMO shortlist, anda dapat mencoba mencoba beberapa soal pada problem set di bab ini. Tapi perlu diketahui bahwa tidak semua ketaksamaan bentuk pecahan dapat diselesaikan dengan cara serupa, ada beberapa diantaranya yang terlalu ‘kuat’ untuk diselesaikan dengan CS. Contoh 2.1.4 (Math Magazine 1634) Jika a , b dan c adalah bilangan riil positif, tentukan bilangan riil k terkecil sehingga ab ac bc + + a + b + 2c 2c b + c + 2b 2b b + c + 2a 2a
≤ k(a + b + c)
dan tentukan kapan terjadi kasus samadengan. Solusi Klaim: jika a,b,c > 0 maka ab ac bc + + a + b + 2c 2c b + c + 2b 2b b + c + 2a 2a
≤ 14 (a + b + c)
yang berarti k = 14 akan memenuhi ketaksamaan, kemudian akan ditunjukkan bahwa k tersebut adalah yang terkecil dengan memperlihatkan adanya kesamaan. Kita akan membuktikan klaim ini, perhatikan bahwa dengan CS engel kita peroleh ab ab + a+c b+c bc bc + b+a c+a ac ac + c+b b+a
≥ a +4bab+ 2c 2c ≥ b + 4cbc+ 2a 2a 4ac ≥ c + a + 2b 2b
Tambahkan ketiga ketaksamaan diatas kita peroleh
ab ac bc 4 + + a + b + 2c 2c b + c + 2b 2b b + c + 2a 2a ab ab bc bc ac ac + + + + + a+c b+c b+a c+a c+b b+a = a+b+c
≤
yang setara dengan ketaksamaan pada soal sehingga klaim terbukti sekarang akan diperiksa apakah k = 14 itu adalah yang terkecil, jelas bahwa jika a = b = c maka kita memperoleh kesamaan sehingga k tersebut memang yang terkecil. Mari kita tinjau lebih lanjut kasus kasus kesamaan, kesamaan, karena kita menggunak menggunakan an CS sebanyak tiga kali maka tanda samadengan berlaku jika dan hanya jika a + c = λ1 = (b ( b + c)
a + b = λ2 = (a + c)
b + c = λ3 = (a + b)
untuk suatu bilangan riil positif λ positif λ1 , λ2 dan λ3 . Jadi tanda samadengan berlaku jika dan hanya jika a = b = c.
30
BAB 2. 2. KETAKSAM KETAKSAMAAN AAN NORM NORM
Contoh 2.1.5 (Moldova 2006) Jika a, b,c > 0 dengan abc = 1 buktikan a+3 b+3 c+3 + + 2 2 (a + 1) (b + 1) (c + 1)2
≥3
Solusi Kita akan kembali menggunakan Lemma yang sama seperti pada soal China China 200 2005, 5, tapi tapi disini disini lemma lemma akan akan dibukt dibuktik ikan an dengan dengan cara cara berbeda berbeda untuk untuk bilangan bilangan riil positif. Lemma: Jika x, y adalah bilangan riil positif maka 1 1 + 2 (x + 1) (y + 1)2
≥ 1 +1 xy
Bukti Lemma: Lemma: Dengan menggunakan CS diperoleh
√
( x+
√x · √xy) xy)2 ≤ (x + y)(1 + xy) xy )
yang ekuivalen dengan 1 (1 + x)2
≥ x x+y
1 1 + xy
dengan cara yang serupa 1 (1 + y )2
≥ y x+y
1 1 + xy
apabila keduanya keduanya ditambahkan ditambahkan 1 1 + (1 + x)2 (1 + y)2
≥ x+y x+y
1 1 + xy
merupakan lemma yang diminta. Kemudian dengan menerapkan Lemma diperoleh 2 2 2 + + 2 2 (a + 1) (b + 1) (c + 1)2
≥ 1 +1 ab + 1 +1 bc + 1 +1 ac
Dengan menambahkan kedua ruas dengan c+1 1 (c+1)2 = c+1 diperoleh a+3 b+3 c+3 + + 2 2 (a + 1) (b + 1) (c + 1)2
a+1 (a+1)2
=
1 a+1
,
b+1 (b+1)2
=
1 b+1
dan
≥ 1 +1 ab + 1 +1 bc + 1 +1 ac + 1 +1 a + 1 +1 b + 1 +1 c
sedangkan sedangkan kondisi abc = 1 menyebabkan bagian sebelah kanan menjadi a b c 1 1 1 + + + + + =3 1+a 1+b 1+c 1+a 1+b 1+c
¨ 2.1.. KETAKS 2.1 KETAKSAMA AMAAN AN CAUCH CAUCHY-S Y-SCHW CHWARZ ARZ (CS), (CS), MIN MINKO KOWSK WSKII DAN DAN H OLDER 31 Contoh 2.1.6 (Vasile Cirtoaje) Untuk bilangan bilangan riil positif positif a,b,c,d. a,b,c,d. Buktikan a b b c c d d a + + + 0 b+c c+d a+d a+b
−
−
−
− ≥
Solusi Melalui aplikasi dari CS-Engel a b b c c d d a + + + b+c c+d a+d a+b a b c+c b c d+d c = + + b+c c+d a+c b+d c+a d+b = + + + 4 b+c c+d a+d a+b 1 1 = ( a + c) + + (b ( b + d) b+c a+d 4 ( a + c) + (b (b + d) a+b+c+d = 0
−
− − −
−
− − −
≥
−d−a+a + d−a−b+b
−
a+d
a+b
− −
1 1 + c+d a+d 4 a+b+c+d
4
4
Teorema berikut merupakan salah satu generalisasi dari Ketaksamaan CauchySchwarz. Teorema 2.1.2 (Ketaksamaan H¨ older) older) Jika λ Jika λ1 , λ2 , , λn adalah bilangan riil positif yang memenuhi λ1 + λ2 + + λn = 1, 1 , maka untuk sebarang bilangan riil positif aij berlaku
···
···
≥ ··· ≥ ··· · · · ≥ · · · ··· n
λi
m
m
n
λi aij
aij
i=1
j =1
j =1
Bukti Misalkan
i=1
λi
m
Ai =
.
aij
j =1
maka berdasarkan AM-GM terboboti kita peroleh a1j a2j λ1 + λ2 + 1 /λ (A1 ) 1 (A2 )1/λ2
anj + λ1 (An )1/λn
Kemudian dengan melakukan operator tusikan
λ1
1/λ Ai i
m j =1 m j =1
=
a1j +λ2 a1j
m j =1 aij
m j =1
aλ1j1 A1
aλ2j2 A2
aλnjn An
pada kedua ruas lalu mensubti-
kita perole p eroleh h
m j =1 a2j + m j =1 a2j
+λn
m j =1 anj m j =1 anj
m
1
A1 A2
An
aij a2j
j =1
anj
32
BAB 2. 2. KETAKSAM KETAKSAMAAN AAN NORM NORM
Sehingga 1 = λ1 + λ2 +
· · · + λn
yang setara dengan
≥ 1
n i=1
m
n
j =1
i=1
aij
Ai
≥ n
λi
m
m
n
λi aij
aij
i=1
j =1
j =1
.
i=1
Contoh 2.1.7 Buktikan bahwa untuk x,y,z adalah bilangan riil positif maka (x2 y 2 z 2 + xyz + 1)2
≤ 98 (x4 + 1)(y 1)(y4 + 1)(z 1)(z 4 + 1)
Solusi oleh ThAzNi (Mathlinker) Pertama - tama kita buktikan dulu Lemma berikut Lemma: Jika t adalah adalah bilangan bilangan riil positif maka berlaku 9 4 (t + 1)3 8
≥ (t6 + t3 + 1)2
bukti lemma Perhatikan bahwa 9(t 9(t4 + 1)3 8(t 8(t6 + t3 + 1)2 = (t 1)4 (t8 + 4t7 + 10 10tt6 + 4t5 2t4 + 4t3 + 10 10tt2 + 4t + 1)
−
−
−
karena dengan AM-GM berlaku t8 + 1 9(t 9(t4 + 1)3
≥ 2t4 maka
− 8(t 8(t6 + t3 + 1)2 ≥ 0
dan lemma telah terbukti. Dengan Dengan lemma lemma tersebut tersebut diperoleh 9(x 9(x4 + 1)3
− 8(x 8(x6 + x3 + 1)2 ≥ 0 9(y 9(y4 + 1)3 − 8(y 8(y 6 + y 3 + 1)2 ≥ 0 9(z 9(z 4 + 1)3 − 8(z 8(z 6 + z 3 + 1)2 ≥ 0.
Hasil perkaliannya 9 4 (x + 1)(y 1)(y 4 + 1)(z 1)(z 4 + 1) 8
≥ [(x [(x6 + x3 + 1)(y 1)(y6 + y3 + 1)(z 1)(z 6 + z 3 + 1)]2/3
Dengan ketaksamaan H¨older older diperoleh [(x [(x6 + x3 + 1)1/3 (y6 + y 3 + 1)1/3 (z 6 + z 3 + 1)1/3 ]2
≥ (x2y2z2 + xyz + 1)2
Ketaksamaan selanjutnya merupakan merupakan salah satu generalisasi dari ketaksamaan Cauchy-Schwarz yang biasa disebut Ketaksamaan Minkowski.
¨ 2.1.. KETAKS 2.1 KETAKSAMA AMAAN AN CAUCH CAUCHY-S Y-SCHW CHWARZ ARZ (CS), (CS), MIN MINKO KOWSK WSKII DAN DAN H OLDER 33 Teorema 2.1.3 (Minkowski) Misalkan r > s adalah bilangan riil yang tidak sama dengan 0, maka untuk bilangan positif aij berlaku
≥ m
s/r
n
1/s
n
r aij
j =1
Latihan 4
1/r
r/s
m
s aij
i=1
i=1
j =1
1. (IMO Shortlist 1993) 1993) Jika a,b,c,d adalah bilangan riil positif, buktikan a b c d + + + b + 2c 2c + 3d 3d c + 2d 2d + 3a 3a d + 2a 2a + 3b 3b a + 2b 2b + 3c 3c
≥ 23
2. (Romania 1997) 1997) Jika a,b,c adalah bilangan riil positif, buktikan a2 b2 c2 + + a2 + 2bc 2bc b2 + 2ac 2ac c2 + 2ab 2ab
ac ab ≥ 1 ≥ a2 +bc2bc + 2 + 2 2bc c + 2ab 2ab c + 2ab 2ab
3. (Canada 2002) 2002) Jika a,b,c adalah bilangan riil positif, buktikan a3 b3 c3 + + bc ac ab
≥a+b+c
4. (Ireland Ireland 1998) 1998) Buktikan bahwa jika a,b,c adalah bilangan riil positif maka 9 a+b+c dan
≤ 2
1 1 1 + + a+b b+c a+c
1 1 1 + + a+b b+c a+c
5. (Iran 1998) 1998) Misalkan x, y,z > 1 dan
≤ 1 2
1 x
+
√x + y + z ≥ √x − 1 +
1 y
+
1 1 1 + + a b c 1 z
= 2. Buktikan
− √ − y
1+
z
1
6. Jika x,y,z bilangan riil positif, buktikan
√ √xyz ) ≤ 3(x 3(x + xy + xyz) 3
√
2 xy 8+ x+y
3
x
x+y 2
x+y+z 3
7.( 7.(IMO 1995) 1995) Jika a,b,c adalah bilangan riil positif yang memenuh memenuhii abc = 1, buktikan 1 1 1 3 + + a3 (b + c) b3 (a + c) c3 (b + a) 2
≥
8.( 8.(USAMO 1997) 1997) Jika x,y,z adalah bilangan riil positif maka buktikan a3
1 1 1 + 3 + 3 3 3 + b + abc b + c + abc c + a3 + abc
1 ≤ abc
34
BAB 2. 2. KETAKSAM KETAKSAMAAN AAN NORM NORM
Hint: Kalikan kedua ruas dengan abc lalu lanjutkan lanjutkan seperti seperti solusi IMO 2001. 9.( 9.(China,????) China,????) Misalkan ri , si , ti , ui , vi adalah bilangan riil tidak kurang dari 1, untuk i = 1, 2, , n dan misalkan R,S,T,U,V, masing-masing merupakan rataan aritmatik (Aritmatic Mean) dari ri , si , ti , ui , vi secara secara berurutan. berurutan. Buktikan n n ri si ti ui vi + 1 RSTUV + 1 ri si ti ui vi 1 RSTUV 1 i=1
···
≤
−
−
10. (IMO 2005) 2005) Misalkan x,y,z adalah bilangan riil positif sehingga xyz Buktikan x5 x2 y 5 y2 z5 z2 + + 0 x5 + y 2 + z 2 x2 + y5 + y2 x2 + y 2 + z 5
−
Hint: Buktikan
2.2
x2 +y 2 +z 2 x5 +y 2 +z 2
−
2
2
2
2
yz +y +z ≤ yz+ x +y +z 2
−
≥ 1.
≥
.
Rearrange Rearrangemen mentt & Ketaksam Ketaksamaan aan Chebishe Chebishev v
Sekarang merupakan saat yang tepat untuk memperkenalkan pengunaan notasi cyc dan sym dalam menulisk menuliskan an ekspresi ekspresi matematik matematik.. Notasi Notasi ini sangat sangat membant mem bantu u kita dalam menulis solusi. Kita mulai dengan notasi sym . MisMisalkan E (a,b,c) a,b,c) adalah fungsi tiga variabel, didefinsikan
E (a,b,c) a,b,c) = E (a,b,c)+ a,b,c)+ E (a,c,b)+ a,c,b)+ E (b,a,c)+ b,a,c)+ E (b,c,a)+ b,c,a)+ E (c,a,b)+ c,a,b)+ E (c,b,a) c,b,a)
sym
Dengan kata lain sym E (a,b,c) a,b,c) merupakan penjumlahan fungsi E (a,b,c) a,b,c) diikuti oleh permutasi dari a , b dan c pada fungsi yang bersangkutan. Kita dapat mengeneralisasikan notasi ini untuk fungsi n variabel E (x1 , x2 , , xn ). Untuk kasus 3 variabel diatas terdapat 6 = 3! suku (permutasi), sedangkan untuk 4 variabel terdapat 4! suku dan seterusnya n! suku untuk n varibel. Untuk notasi cyc didefinisikan sebagai
···
E (a,b,c) a,b,c) = E (a,b,c) a,b,c) + E (b,c,a) b,c,a) + E (c,a,b) c,a,b)
cyc
Dengan kata lain cyc merupakan setengah dari penjumlahan siklik dimana posisi a, b dan c berganti menurut a menjadi b , b menjadi c dan c menjadi a. Untuk Untuk kasus kasus 3 variabel variabel terdapat terdapat 12 (3!) suku. Generalisa Generalisasi si untuk n variabel dapat dikembangkan secara serupa. Contoh 2.2.1 Jika kita ingin menulis xy + yz + xz maka dapat ditulis
cyc
xy
atau
1 xy 2 sym
2.2. REARRANGEMEN REARRANGEMENT T & KETAKSAMAA KETAKSAMAAN N CHEBISHEV CHEBISHEV
35
Contoh 2.2.2 Ketaksamaan Nesbitt pada contoh 1.5.2 dapat ditulis
cyc
a b+c
≥ 32
a b+c sym
atau
≥3
Contoh 2.2.3 Ekspansi dari (x + y + z )2 (xy + yz + xz) xz ) dapat dituliskan
x3 y + 5
sym
cyc
x2 yz +
x2 y 2
sym
Seringkali didalam menulis solusi kita menggunakan frase ” Tanpa kehilangan keumuman ” atau dalam bahwa inggris di singkat WLOG yaitu ”Without Loss of Generality”, dikarenakan untuk menyelesaikan suatu soal kita cukup mempertim mempertimbangk bangkan an sedikit sedikit kasus. kasus. Perhatik Perhatikan an bahwa bahwa sering kali berguna berguna bagi kita didalam menyelesaikan soal pertidaksamaan berbentuk simetris atau siklis dengan mengasumsikan a b c atau x1 x2 xn (Ingat, penalaran ini telah kita gunakan gunakan sebelumnya sebelumnya pada solusi soal IMO 2006). 2006). Misalkan Misalkan pada ketaksamaan Nesbitt diperbolehkan WLOG a b c karena jika
≥ ≥
f (a,b,c) a,b,c) =
≥ ≥ ··· ≥ ≥ ≥
a b c + + b+c a+c a+b
dapat dilihat bahwa tidak masalah mana yang terbesar diantara a, b atau c, nilai f tetap sama, dapat dilihat f (1 f (1,, 2, 3) = f (1 f (1,, 3, 2) = f (2 f (2,, 1, 3) = f (2 f (2,, 3, 1) = f (3 f (3,, 1, 2) = f (3 f (3,, 2, 1) = 17 . Faktanya semua ketaksamaan yang simetris berlaku 10 seperti ini, sedangkan pada ketaksamaan siklis, pertama - tama kita lakukan pembuktian dengan syarat a b c kemudian lakukan juga pembuktian dengan syarat a c b. Hal ini tentunya tentunya sangat berguna, berguna, karena karena untuk membuktikan suatu ketaksaman yang simetrik kita cukup membuktikan satu kasus saja daripada menuliskan bukti kasus per kasus yang pada dasarnya sebenarnya mempunyai jalan yang sama.
≥ ≥
≥ ≥
Contoh 2.2.4 Adalah suatu keputusan yang tidak tepat bagi kita untuk mengambil langkah WLOG a b c dalam membuktikan ketaksamaan
≥ ≥
a2 b = a2 b + b2 c + c2 a
cyc
≥3
dengan syarat abc = 1 dan a , b , c bilangan riil positif. Karena jika f ( f (a,b,c) a,b,c) = 1 17 1 2 2 2 a b + b c + c a maka f 1, 2 , 2 = 5 = 4 = f 1, 2, 2 .
Berikutny Berikutnyaa akan akan diberikan diberikan contoh contoh untuk untuk menunjukk menunjukkan an bagaimana bagaimana notasi notasi dan dapat membantu dalam penulisan solusi cyc sym Sesuatu yang akan kita pelajari pada bahasan selanjutnya sedikit berbeda dengan yang sebelumnya, Rearrangement sebenarnya bukanlah benar - benar ketaksamaan seperti halnya AM-GM, CS atau ketaksamaan Jensen, ini lebih kepada metode yang digunakan untuk menyelesaikan sebuah persoalan ketaksamaan. Sedangkan dari Rearrangement dapat diturunkan sebuah ketaksamaan yang sering disebut ketaksamaan Chebishev. Untuk memulai bahasan kita tentang Rearrangement, mari kita tinjau contoh sederhana berikut.
36
BAB 2. 2. KETAKSAM KETAKSAMAAN AAN NORM NORM
Contoh 2.2.5 Sebuah mangkok berisi 24 keping uang logam yang terdiri dari 8 Keping uang 1000 , 8 Keping uang 500 dan 8 keping uang 100. Seseorang boleh melakukan pengambilan uang tersebut dengan cuma - cuma tapi dengan syarat: - Pengambilan dilakukan tiga kali. - Pada pengambilan pertama hanya boleh diambil 4 keping. - Pada pengambilan kedua hanya boleh diambil 2 keping. - Pada pengambilan ketiga hanya boleh diambil 6 keping. Bagaimana cara pengambilan agar diperoleh a. Uang maksimum? b. Uang minimum? Solusi a. Tentu saja untuk mendapatkan banyak uang strateginya adalah dengan mengambil uang yang bernilai besar yaitu 1000 sebanyak mungkin, dan kemudian kemudian diikuti yang kedua kedua terbesar. Jadi pada pengambilan pengambilan pertama kita ambil uang bernilai 500 yaitu berdasarkan syarat 4 keping, pada pengambilan kedua ambil 2 keping uang 100 dan pada pengambilan yang ketiga adalah 6 keping uang 1000. Jadi kita dapatkan 4(500) + 2(100) + 6(1000) = 8200 Pengambilan dengan cara lain tapi tetap menurut syarat diatas akan membuat uang yang dihasilkan dihasilkan lebih kecil dari 8200. b. Untuk mendapatkan uang minimum dapat diperoleh dengan cara 4(500) + 2(1000) + 6(100) = 4600 Jadi bagaimanapun cara pengambilan (dengan syarat diatas), uang yang diperoleh pasti lebih dari 4600. Sekarang kita akan menyatakan Rearrangement yang diambil dari contoh diatas. Teorema 2.2.1 Misalkan terdapat dua pasangan terurut (a1 , a2 , , an ) dan (b1 , b2 , , bn ) dengan ai dan bi adalah adalah bilangan bilangan riil untuk i = 1, 2, , n. Jika a1 a2 an dan b1 b2 bn , maka hasil kali titik dengan nilai maksimum adalah S M + an bn M = a1 b1 + a2 b2 +
··· ≥ ≥ · ·· ≥
··· ···
≥ ≥ · ·· ≥
···
dan hasil kali titik dengan nilai minimum adalah S m = a1 bn + a2 bn−1 +
· · · + anb1 Dengan kata lain jika pasangan terurut (a1 , a2 , · · · , an ) dikalikan titik dengan permutasi dari pasangan terurut (b1 , b2 , · · · bn ) katakanlah (bσ(1) , bσ , · · · , bσ ) maka berlaku S m ≤ S p ≤ S M M . Dengan tanda sama dengan terjadi jika dan hanya jika a1 = a2 = · · · = an atau b1 = b2 = · · · = bn 2
n
37
2.2. REARRANGEMEN REARRANGEMENT T & KETAKSAMAA KETAKSAMAAN N CHEBISHEV CHEBISHEV
Bukti Tanpa kehilangan keumuman, cukup bagi kita untuk mempertimbangkan apabila pengurutan pada ekspresi S p diperoleh dengan menukar dua buah bilangan riil ai dan aj . Misalkan ai aj dan bi bj , dan
≥ ≥ S M M = a1 b1 + a2 b2 + · · · + ai bi + · · · + aj bj + · · · + an bn S p = a1 b1 + a2 b2 + · · · + aj bi + · · · + ai bj + · · · + an bn maka S p − S M )(aj − ai ) ≤ 0, jadi S p ≤ S M M = bi (aj − ai ) + bj (ai − aj ) = (bi − bj )(a M . Untuk melihat kapan kesamaan terjadi, perhatikan bahwa ( bi − bj )(a )(aj − ai ) = 0 jika jika dan hanya hanya jika jika ai = aj atau bi = bj . Deng Dengan an cara cara yang yang sama sama dapa dapatt dibuktikan S m ≤ S p .
Contoh 2.2.6 (International Mathematical Olympiad 1975) Diberikan Diberikan bilangan riil x1 x2 xn dan y1 y2 yn . Jika Jika (z1 , z2 , , zn ) adalah permutasi dari (y1 , y2 , , yn ). Buktikan bahwa
≤
≤ ·· · ≤
≤ ≤ ·· · ≤
···
n
n
i=1
Solusi Karena (z (z1 , z2 ,
···
(xi − yi )2 ≤
(xi
i=1
− zi )2
· · · , zn) adalah permutasi dari (y ( y1 , y2 , · · · , yn ) maka n
n
x2i
=
i=1
zi2
i=1
Dengan demikian n
n
(xi
i=1
− yi)2 ≤
n
(xi
i=1
− zi )2 ⇐⇒
n
≥ xi yi
i=1
xi zi
i=1
yang dapat diperoleh dari rearrangement. Contoh 2.2.7 (International Mathematical Olympiad 1983) Misalkan a Misalkan a , b dan c adalah panjang sisi - sisi sebuah segitiga. Buktikan bahwa a2 b(a
− b) + b2c(b − c) + c2a(c − a) ≥ 0
Solusi Perhatikan bahwa ketaksamaan diatas siklik, jadi kita cukup membuktikan untuk kasus dimana a b c dan a c b. Karena a, b, dan c adalah panjang panjang sisi pada segitiga maka maka berlaku
≥ ≥
a+b>c
≥ ≥
b+c>a
a+c> b
Misalkan a b c maka b(c + a b) a(b + c a) = (a b)(a )(a + b c) 0 sehingga b(c + a b) a(b + c a). Dengan Dengan cara yang sama kita kita dapatkan dapatkan c(a + b c) b(c + a b), jadi
≥ ≥ − ≥ − ≥ −
c(a + b
− − −
−
−
− c) ≥ b(c + a − b) ≥ a(b + c − a)
− ≥
38
BAB 2. 2. KETAKSAM KETAKSAMAAN AAN NORM NORM
Juga perhatikan bahwa karena a
≥ b ≥ c maka 1 1 1 ≥ ≥ c b a
Jadi berdasark berdasarkan an Rearrangem Rearrangement ent pada b), a(b + c a)) kita perloleh
−
1 c( a + b c
1 1 1 c, b, a
dan (c (c(a + b
− c), b(c + a −
− c) + 1b b(c + a − b) + a1 a(b + c − a) = a + b + c
adalah ekspresi yang paling maksimum, dan juga ekspresi lain yaitu 1 c(a + b b harus memenuhi 1 c( a + b b
− c) + a1 b(c + a − b) + 1c a(b + c − a)
− c) + a1 b(c + a − b) + 1c a(b + c − a) ≤ a + b + c
kalikan kedua ruas dengan abc diperoleh ac2 (a + b
− c) + b2c(c + a − b) + a2b(b + c − a) ≤ abc( abc(a + b + c)
yang disederhanakan mencjadi ac2 (a
− c) + b2c(a − b) + a2(b − a) ≤ 0 ⇐⇒ a2b(a − b) + b2c(b − c) + c2a(c − a) ≥ 0 Untuk kasus a ≥ c ≥ b dengan cara yang serupa kita dapatkan b(c + a − b) ≥ c(a + b − c) ≥ a(b + c − a) selanjutnya kita dapat menggunakan cara yang serupa dengan kasus pertama. Misalkan terdapat pasangan terurut (a ( a1 , a2 , , an ) dan (b ( b1 , b2 , gan a1 a2 an dan b1 b2 bn dan misalkan
≥ ≥···≥ (x1 , x2 , · · · , xn )
···
≥ ≥···≥ ··· (xn , x1 , · · · , xn−1 )
adalah n buah permutasi siklis berbeda dari (b ( b1 , b2 , Rearrangement berlaku
(x2 , x3 ,
· · · , bn) den-
· · · , x1 )
· · · , bn), maka berdasarkan
(a1 b1 + a2 b2 + (a1 b1 + a2 b2 +
· · · + anbn) ≥ a1x1 + a2x2 + · · · + anxn · · · + anbn) ≥ a1xn + a2x1 + · · · + anxn−1 ······ ······ (a1 b1 + a2 b2 + · · · + an bn ) ≥ a1 x2 + a2 x3 + · · · + an x1 Jumlahkan semua ketaksamaan diatas kita peroleh n(a1 b1 + a2 b2 +
· · · + anbn) ≥ a1(x1 + xn + · · · + x2) + a2(x2 + x1 + · · · + x3) + · · · + an (xn + xn−1 + · · · + x1 ) = (a ( a1 + a2 + · · · + an )(x )(x1 + x2 + · · · + xn )
39
2.2. REARRANGEMEN REARRANGEMENT T & KETAKSAMAA KETAKSAMAAN N CHEBISHEV CHEBISHEV Karena x1 + x2 +
· · · + xn = b1 + b2 + · · · + bn maka (a1 + a2 + · · · + an )(b )(b1 + b2 + · · · + bn ) (a1 b1 + a2 b2 + · · · + an bn ) ≥ n
Dengan tanda membuktikan
≥ berubah menjadi ≤ pada kasus b1 ≤ b2 ≤ · · · ≤ bn , kita telah
Teorema 2.2.2 Jika ( Jika (a1 , a2 , rut dari bilangan riil. Jika a1
· · · , an) dan ( dan (b1 , b2 , · · · , bn ) adalah pasangan teru≥ a2 ≥ · · · ≥ an dan b1 ≥ b2 ≥ · · · ≥ bn maka (a1 + a2 + · · · + an )(b )(b1 + b2 + · · · + bn ) (a1 b1 + a2 b2 + · · · + an bn ) ≥ n
Jika a1
≥ a2 ≥ · · · ≥ an dan b1 ≤ b2 ≤ · · · ≤ bn maka (a1 + a2 + · · · + an )(b )(b1 + b2 + · · · + bn ) (a1 b1 + a2 b2 + · · · + an bn ) ≤ n
Contoh 2.2.8 (Romania 2005, Cezar Lupu) Buktikan bahwa untuk sebarang sebarang bilangan riil positif a , b dan c berlaku b+c a+c a+b + 2 + 2 a2 b c Ketaksamaan diatas simetrik, WLOG a 1 a
≤ 1b ≤ 1c
≥ 2
1 1 1 + + a b c
≥ b ≥ c maka
b+c a
≤ a +b c ≤ a +c b
Dengan ketaksamaan Chebisev diperoleh b+c a+c a+b + 2 + 2 a2 b c
≥
1 3
b+c a+c a+b + + a b c
Dan dengan CS engel dan ketaksamaan ( a + b + c)2 (b + c)2 (a + c)2 (a + b)2 + + a(b + c) b(a + c) c(a + b) jadi 1 3
1 1 1 + + a b c
≥ 3(ab 3(ab+ + ac+ ac + bc) bc) kita peroleh 2
2(a + b + c) ≥ 2(a ≥6 ab + ac + bc
b+c a+c a+b + + a b c
≥
2
Dengan demikian b+c a+c a+b + 2 + 2 a2 b c
≥ ≥
1 3 2
b+c a+c a+b + + a b c 1 1 1 + + . a b c
1 1 1 + + a b c
40
BAB 2. 2. KETAKSAM KETAKSAMAAN AAN NORM NORM
Contoh 2.2.9 (Fajar Yuliawan ) Jika xi > 0, xi buktikan n n
xi 2 + nn−1
i=1
xi n (1
i=1
∈
R dan
− xi ) ≥ 1
n i=1 xi
= 1
Solusi Kita akan membuktikan generalisasi soal tersebut yaitu n
n
xi 2 + nm−1
i=1
xi m (1
i=1
− xi ) ≥ 1
dengan in=1 xi = 1 dan m = 0, 1, 2, . Soal tersebut tersebut akan akan dibuktikan dibuktikan mengmenggunakan dua lemma berikut: Lemma 1 Jika in=1 xi = 1 dan xi 0 dengan x1 x2 xn maka
··· ≥ ≥ ≥···≥ 1 + nn−1 x1 n − nn−1 x1 n+1 ≥ 1 + nn−1 x2 n − nn−1 x2 n+1 ≥ ··· ≥ 1 + nn−1xnn − nn−1xnn+1
Bukti Lemma 1: Kita akan membuktikan
− xin+1 ≥ xi+1n − xi+1n+1 i = 1,1 , 2, · · · , n − 1 Karena xi ≥ xi+1 maka xi n−1 − xi+1 n−1 ≥ 0 dan xi n − xi+1 n ≥ 0, karena n i=1 xi = 1 dan xi ≥ 0 maka 1 − (xi + xi+1 ) ≥ 0, jadi (xi n − xi+1 n ) ≥ (xi + xi+1 )(x )(xi n − xi+1 n ) −1 ) = xi n+1 − xi+1 n+1 + xi xi+1 (xi n−1 − xin+1 ≥ xin+1 − xi+1n+1 Jadi xi n − xi+1 n ≥ xi n+1 − xi+1 n+1 yang setara dengan xi n − xi n+1 ≥ xi+1 n − xi+1 n+1 xi n
Kalikan dengan nn−1 kemudian tambahkan dengan 1 kita dapatkan 1 + nn−1 xi n
− nn−1xin+1 ≥ 1 + nn−1xi+1n − nn−1xi+1n+1
Lemma 2 Jika xi 0 dengan positif m > 2 berlaku
≥
n
n i=1 xi
n
2
m 1
xi + n −
i=1
= 1 maka untuk setiap bilangan bulat n
m
xi (1
i=1
− xi )
≥ i=1
n
2
m 3
xi + n −
xi m−2 (1
i=1
− xi )
Bukti Lemma 2: Pertama - tama tulis n
i=1
n
2
m 1
xi + n −
i=1
n
m
xi (1
− xi) =
i=1
xi 2 (1 + nm−1 xi m−2
− nm−1xi m−1)
41
2.2. REARRANGEMEN REARRANGEMENT T & KETAKSAMAA KETAKSAMAAN N CHEBISHEV CHEBISHEV WLOG x1 Lemma 1
≥ x2 ≥ ··· ≥ xn maka x12 ≥ x22 ≥ ··· ≥ xn2 dan berdasarkan
1 + nn−1 x1 n
− nn−1x1n+1 ≥ 1 + nn−1x2n − nn−1x2n+1 ≥ ··· ≥ 1 + nn−1xnn − nn−1xnn+1
Jadi berdasarkan ketaksamaan Chebishev dan ketaksamaan RMS-AM n 1 1 n ( i=1 xi ) = n diperoleh
n
n
n 2 i=1 xi
≥
− − ≥ − − ≥ − − 2
m 1
xi m (1
xi + n −
i=1
xi )
i=1
n
xi 2 (1 + nm−1 xi m−2
=
nm−1 xi m−1 )
i=1
1 n
n
n
xi 2
n + nm−1
i=1
n
2
m 2
xi + n −
n
xi
i=1 n
2
xi m−2
i=1 n
xi 2 + nm−3
i=1 n
xi m−1
i=1
xi m−2
xi m−1
i=1
n
xi 2 + nm−3
=
xi m−1
i=1
n
=
xi m−2
i=1
xi m−2 (1
xi )
i=1
sehingga sehingga Lemm Lemmaa 2 terbukti. terbukti. Sekarang Sekarang kita akan akan mem membukti buktika kan n soal tersebut, tersebut, definsikan fungsi n
f ( f (m) =
n
2
m 1
xi + n −
i=1
xi m (1
i=1
− xi )
Kita akan membuktikan yang lebih umum yaitu f ( f (m) Perhatikan Perhatikan bahwa n
f (0) f (0) =
xi 2 +
i=1
n
karena
n
i=1
≥1
Jika m genap maka berdasarkan Lemma 2 f ( f (m)
≥ f ( f (m − 2) ≥ · · · ≥ f (0) f (0) ≥ 1
Jika m ganjil maka berdasarkan lemma 2 f ( f (m)
···
≥ 1 untuk m = 0, 1, 2, · · · . n
−1 ≥1 f (1) f (1) = 1
m = 0, 0 , 1, 2,
≥ f ( f (m − 2) ≥ · · · ≥ f (1) f (1) ≥ 1 Subtitusikan m = n maka f ( f (n) ≥ 1 dan soal terbukti.
xi 2
≥ n1
42
BAB 2. 2. KETAKSAM KETAKSAMAAN AAN NORM NORM
Contoh Contoh 2.2.10 (Michael (Michael Rozenberg) Rozenberg) Jika a, b dan c adalah bilangan riil positif. positif. Buktikan a2 + 2bc 2bc b2 + 2ac 2ac c2 + 2ab 2ab + + b+c a+c a+b Solusi WLOG a
≥ 32 (a + b + c)
≥ b ≥ c maka 1 2(b 2(b + c)
1 1 ≥ 2(a ≥ 2(a + c) 2(a 2(a + b)
dan juga a
≥b ⇔ ⇔ ⇔
a(2a (2a 7c) b(2b (2b 7c) 2a2 + 4bc 4bc 3ac 2b2 + 4ac 4ac 3bc 2 2a + 4bc 4bc 3ac 3ab 2b2 + 4ac 4ac
−
≥ − − ≥ − − ≥
−
− 3bc − 3ab
dengan cara yang serupa 2b 2 b2 + 4ac 4ac
− 3bc − 3ab ≥ 2c2 + 4ab 4ab − 3ac − 3bc. bc. Jadi 2a2 + 4bc 4bc − 3ac − 3ab ≥ 2b2 + 4ac 4ac − 3bc − 3ab ≥ 2c2 + 4ab 4ab − 3ac − 3bc
sehingga dengan Rearrangement kita dapatkan 2a2 + 4bc 4bc 3ac 3ab 2b2 + 4ac 4ac 3bc 3ab 2c2 + 4ab 4ab 3ac + + 2(b 2(b + c) 2(a 2(a + c) 2(a 2(a + b) 1 2 1 1 1 (a + b2 + c2 ab ac bc) bc) + + . 3 b+c a+c a+b
−
−
−
− − −
−
−
− 3bc ≥
Karena a2 + b2 + c2
≥ ab + ac + bc maka 2a2 + 4bc 4bc − 3ac − 3ab 2b2 + 4ac 4ac − 3bc − 3ab 2c2 + 4ab 4ab − 3ac − 3bc ≥0 + + 2(b 2(b + c)
2(a 2(a + c)
2(a 2(a + b)
sedangkan ketaksamaan ini setara dengan
2(a 2(a2 + bc) bc) 2(b 2(b + c)
−
3 a + 2
2(b 2(b2 + ac) ac) 2(a 2(a + c)
−
3 b + 2
2(c 2(c2 + ab) ab) 2(a 2(a + b)
−
≥
3 c 2
0
sehingga seperti yang diinginkan kita peroleh a2 + 2bc 2bc b2 + 2ac 2ac c2 + 2ab 2ab + + b+c a+c a+b
≥ 32 (a + b + c).
Contoh 2.2.11 (International Mathematical Olympiad 2005) Misalkan x,y,z adalah bilangan riil positif sehingga xyz 1. Buktikan
≥
x5 x2 y 5 y2 z5 z2 + + x5 + y2 + z 2 x2 + y 5 + z 2 x2 + y2 + z 5
−
−
−
≥0
2.2. REARRANGEMEN REARRANGEMENT T & KETAKSAMAA KETAKSAMAAN N CHEBISHEV CHEBISHEV
43
Solusi Kita mulai dengan membuktikan kedua buah lemma. Lemma 1: Untuk x,y,z bilangan bilangan riil positif p ositif dengan xyz 1 berlaku
≥
x5 y5 z5 + + x5 + y 2 + z 2 x2 + y5 + z 2 x2 + y 2 + z 5
≥1
Bukti Lemma 1: Dengan menggunakan CS Engel kita peroleh
≥
x6 y6 z6 + + x6 + xy 2 + xz 2 yx 2 + y 6 + xz 2 x2 z + y 2 z + z 6 (x3 + y3 + z 3 )2 x6 + y6 + z 6 + x(y2 + z 2 ) + y (x2 + z 2 ) + z (x2 + y 2 )
Jadi cukup diperiksa apakah 2(x 2(x3 y3 + z 3 x3 + y3 z 3 )
≥ x(y2 + z2) + y(x2 + z2) + z(x2 + y2)
dengan kondisi xyz
≥ 1 kita tinggal membuktikan x2 y 2 z 2 x2 y2 z 2 ≥ x(y2 + z2) + y(x2 + z2) + z(x2 + y2) 2 + + x y z Untuk membuktikan ini WLOG x ≥ y ≥ z maka xy xz yz ≥ ≥ xy ≥ xz ≥ yz dan z y x
maka berdasarkan Rearrangement
≥
x2 y 2 z 2 x2 y2 z 2 + + x y z
xy xz + z
xz y
yz +
yz xy x
= x2 y + z 2 x + y 2 z
≥
x2 y 2 z 2 x2 y2 z 2 + + x y z
xy yz + z
xz y
xy +
yz xz x
= y 2 x + x2 z + z 2 y
menambahkan dua ketaksamaan diatas akan melengkapi bukti lemma 1. Lemma 2: Untuk bilangan riil x, y dan z dengan xyz 1 berlaku
≥
x2 y2 z2 + + x5 + y 2 + z 2 x2 + y5 + z 2 x2 + y 2 + z 5
≤1 dengan menggunakan ketaksamaan Cauchy dan kondisi xyz ≥ 1 kita peroleh 2
2
2 2
(x + y + z )
2
1 = x5/2 + x2 + y2 x 1 (x5 + y 2 + z 2 ) + y2 + z 2 x
≤ ≤
√
(x5 + y 2 + z 2 ) yz + y 2 + z 2
44
BAB 2. 2. KETAKSAM KETAKSAMAAN AAN NORM NORM
jadi
x2 x5 + y2 + z 2
2
2
2
≤ x(x(2yz++y2y++z2z)2) .
dengan cara yang serupa diperoleh y2 y5 + x2 + z 2
2
2
2
≤ y(x(2xz++y2x++z2z)2)
z2 x2 + y 2 + z 5
dan
2
2
2
≤ z(x(2xy++y2x++z2y)2)
tambahkan semua ketaksamaan ini diperoleh x2 y2 z2 + + x5 + y2 + z 2 x2 + y 5 + z 2 x2 + y 2 + z 5 2(x 2(x2 y2 + x2 z 2 + y 2 x2 ) + x2 yz + xy 2 z + xyz 2 (x2 + y 2 + z 2 )2 2(x 2(x2 y2 + x2 z 2 + y 2 x2 ) + x2 yz + xy 2 z + xyz 2 = x4 + y 4 + z 4 + 2(x 2(x2 y2 + x2 z 2 + y 2 x2 )
≤
Jadi tinggal dibuktikan 2(x 2(x2 y 2 + x2 z 2 + y 2 x2 ) + x2 yz + xy2 z + xyz 2 x4 + y4 + z 4 + 2(x 2(x2 y 2 + x2 z 2 + y 2 x2 )
≤1
yang setara dengan x2 yz + xy 2 z + xyz 2
≤ x4 + y4 + z4
ketaksamaan ini dapat dibuktikan dengan langkah berikut x4 + y 4 + z 4
x2 y 2 + x2 z 2 + y2 z 2 x2 y 2 + x2 z 2 y 2 z 2 + x2 z 2 = + 2 2
≥ ≥
x2 yz + xy 2 z + xyz 2
+
y 2 z 2 + x2 y 2 2
sehingga Lemma 2 Terbukti. Kurangkan ketaksamaan pada lemma 1 dengan ketaksamaan pada lemma 2 kita telah menyelesaikan soal IMO ini.
Latihan 5 1. Jika a, b dan c adalah panjang sisi - sisi pada segitiga tidak degenerate, buktikan 1 1 1 1 1 1 + + + + a+b c b+c a a+c b a b c 2. Jika x1 , x2 , , xn adalah bilangan riil positif dengan x1 x2 xn = 1 dan bilangan riil s dan t sehingga s t buktikan
···
−
−
− ≥
≥ x1 s + x2 s + · · · + xn s ≥ x1 t + x2 t+ · · · + xn t
···
2.2. REARRANGEMEN REARRANGEMENT T & KETAKSAMAA KETAKSAMAAN N CHEBISHEV CHEBISHEV
45
3.(Vasile Cirtoaje) Misalkan a,b,c adalah bilangan riil positif, buktikan buktikan
√a2 + bc √b2 + ac √c2 + ab b+c
+
4.(IMO 1978) Misalkan a1 , a2 , tikan
a+c
+
a+b
≥
√
3 2 2
· · · , an adalah bilangan riil yang berbeda, bukn
ai i2
n
≥ i=1
i=1
1 i
5.(IMO 1964) Misalkan a,b, dan c adalah panjang sisi - sisi sebuah segitiga, buktikan bahwa a2 (b + c
− a) + b2(a + c − b) + c2(a + b − c) ≤ 3abc
6.(German 6.(Germany y 2005) Untuk bilangan riil positif a,b,c dengan a + b + c = 1 buktikan 1+a 1+b 1+c b c a + + 2 + + 1 a 1 b 1 c a b c
−
−
−
≤
46
BAB 2. 2. KETAKSAM KETAKSAMAAN AAN NORM NORM
Bab 3
Penjumlahan Simetrik dan Siklis 3.1
Norma Normalis lisasi asi dan Su Subt btitu itusi si
Beberapa materi yang telah kita pelajari sebelumnya memang berhasil menyelesaikan banyak banyak persoalan pertidaksamaan, materi - materi tersebut dapat dikatakan materi standar dan masih belum cukup untuk menyelesaikan soal - soal yang tingkat kesulitannya tinggi, apalagi seiring berjalannya waktu soal - soal pertidaksamaan tidak menjadi lebih mudah, pembaca mungkin menyadari bahwa banyak soal olimpiade dari mancanegara yang mudah dalam selang waktu 90an. Pertama Pertama - tama kita akan akan belajar tentang tentang normalisasi. normalisasi. Perhatik Perhatikan an kembali kembali Ketaksamaan Nesbitt, yang menyatakan: Untuk a,b,c > 0 a b c 3 + + . b+c a+c a+b 2 Membuktikan ketaksamaan ini sama saja dengan membuktikan ketaksamaan: Jika a,b,c > 0 dengan a + b + c = 1
≥
a b c + + b+c a+c a+b
≥ 32
karena jika a + b + c = k dan subtitusi a = kx, kx, b = kx dan c = kx memberikan kx ky kz + + ky + kz kx + kz kx + ky
≥ 32
dengan kx + ky + kz = k. Dengan kata lain untuk membuktikan membuktikan ketaksamaan Nesbitt dapat dengan cara membuktikan x y z + + y+z x+z x+y dengan kondisi x + y + z = 1. 47
≥ 32
48
BAB 3. PENJUMLAHAN PENJUMLAHAN SIMETRI SIMETRIK K DAN SIKLIS SIKLIS
Perubaha Perubahan n variabel variabel (a,b,c) a,b,c) menj menjad adii (x,y,z) x,y,z) tidak tidak lebih lebih dari dari perubah perubahan an nama variabe saja (kita katakan disini dummy variable ), jadi tidak masalah jika kita tetap menuliskan dalam variabel (a,b,c (a,b,c)) yaitu : Untuk a,b,c > 0 dengan a + b + c = 1 maka a b c + + b+c a+c a+b
≥ 32
Secara tidak langsung kita telah menyatakan ”WLOG a + b + c = 1”. Suatu Suatu hal yang yang mem memungkin ungkinka kan n hal ini pada ketaksamaan ketaksamaan nesbitt adalah karena pada ketaksamaan tersebut subtitusi a = kx, kx , b = ky, ky , c = kz berujung pada hilangnya k . Ketaksamaan yang seperti ini kita katakan homogen. Setiap ketaksamaan dapat kita tulis ke bentuk f ( f (x1 , x2 , , xn ) 0, misalkan pada ketaksamaan Nessbit kita dapat menuliskan
···
f ( f (a,b,c) a,b,c) = dan f ( f (a,b,c) a,b,c)
a b c + + b+c a+c a+b
≥
− 32
≥ 0. Ketaksamaan tersebut dikatakan homogen jika f ( f (λx1 , λx2 , · · · , λxn ) = f ( f (x1 , x2 , · · · , xn )
untuk sebarang bilangan riil λ. Tambahan lagi tidak diharuskan bagi kita untuk mengasumsikan a + b + c = 1, dengan leluasa pada ketaksamaan homogen kita boleh mengasumsikan ab + ac + bc = 1, a2 + b2 + c2 = 1, abc = 1 atau lebih banyak banyak lagi. Terlebih erlebih lagi konstanta konstanta yang dipilih tidak harus angka angka 1, kita boleh mengasums mengasumsik ikan an a + b + c = r untuk nilai r yang tergantung pada soal. Misalkan jika a,b,c > 0 dan ketaksamaannya homogen, kita dapat mengasumsikan a + b + c = 3, tapi tentu saja tidak diperbolehkan untuk mengasumsikan a + b + c = 1. Setelah mengasumsikan sebuah syarat, maka kita akan mendapatkan sebuah ketaksamaan yang tidak homogen lagi. Kita rangkum apa yang telah dijelaskan diatas pada contoh berikut
−
Contoh 3.1.1 (Ketaksamaan Nesbitt) Jika a,b,c > 0 maka a b c + + b+c a+c a+b
≥ 32
Solusi Karena ketaksamaan tersebut homogen maka WLOG a + b + c = 1 dengan 0 < a, a, b, c < 1. Sehingga ketaksamaan diatas dapat ditulis dalam bentuk a 1
−a
+
b 1
−b
+
c
3 ≥ 1−c 2
Untuk membuktikan ketaksamaan diatas kita gunakan Lemma berikut Lemma: Jika x adalah bilangan bilangan riil dengan x < 1 maka berlaku x
9x − 1 ≥ 1−x 4
49
3.1. NORMALISASI NORMALISASI DAN SUBTITU SUBTITUSI SI Bukti Lemma: Lemma: Ketaksamaan akan diperoleh melalui rutunan langkah berikut (3x (3x
− 1)2 ≥ 0 ⇔ ⇔ ⇔ ⇔
9x2 + 4x 4x + 1 10 10x x 2 4x 9x + 10x 10x 1 4x (9x (9x 1)(1 x) x 9x 1 1 x 4
≥
≥− − ≥ − − − ≥ −
sehingga Lemma terbukti. Dengan mengaplikasikan lemma tersebut kita dapatkan a 1
−a
+
b 1
−b
+
c
9a − 1 9b − 1 9c − 1 ≥ + + 1−c 4 4 4 9(a 9(a + b + c) − 3 = 4
3 = . 2 Contoh 3.1.2 (USAMO 2003) Misalkan a,b,c adalah adalah bilangan bilangan riil positif positif,, buktikan (2a (2a + b + c)2 (a + 2b 2b + c)2 (a + b + 2c 2c)2 + + 8 2a2 + (b (b + c)2 2b2 + (a (a + c)2 2c2 + (a (a + b)2
≤
Solusi Karena ketaksamaan tersebut homogen maka WLOG a + b + c = 3, jadi ketaksamaan diatas berubah menjadi (a + 3)2 (b + 3)2 (c + 3)2 + + 2a2 + (3 a)2 2b2 + (3 b)2 2c2 + (3 c)2
−
−
−
Kita bentuk Lemma berikut Lemma: Jika x adalah adalah bilangan bilangan riil dengan dengan x (x + 3)2 2x2 + (3 x)2
−
≤ 8.
≥ − 34 maka berlaku
≤ 43 (x + 1)
Bukti Lemma: Lemma: Ketaksamaan akan diperoleh melalui rutunan langkah berikut (x
− 1)2(4x (4x + 3) ≥ 0 ⇔ ⇔ ⇔ ⇔ ⇔
4x3 5x2 2x + 3 0 4x3 4x2 + 4x 4x + 12 x2 + 6x 6x + 9 2 2 4(x 4(x + 1)(x 1)(x 2x + 3) x + 6x 6x + 9 2 4 x + 6x 6x + 9 (x + 1) 2 3 3(x 3(x 2x + 3) 4 (x + 3)2 (x + 1) 3 2x2 + (3 x)2
− −
−
≥ ≥ ≥
−
≥ ≥
−
−
50
BAB 3. PENJUMLAHAN PENJUMLAHAN SIMETRI SIMETRIK K DAN SIKLIS SIKLIS
membuktikan membuktikan Lemma. Dengan menggunakan Lemma kita dapatkan (a + 3)2 (b + 3)2 (c + 3)2 + + 2a2 + (3 a)2 2b2 + (3 b)2 2c2 + (3 c)2
−
−
4 (a + b + c + 3) 3 = 8.
≤
−
Sesuatu yang tampak tidak natural pada dua bukti diatas terletak pada Lemma yang digunakan. Pada contoh 1.8.1 kita menggunakan menggunakan x
9x − 1 ≥ 1−x 4
pada contoh 1.8.2 kita menggunakan
(x + 3)2 2x2 + (3 x)2
−
≤ 43 (x + 1)
bagaimana bagaimana kedua kedua Lemm Lemmaa ini bisa terpikirk terpikirkan? an? Untuk Untuk kasus pada contoh ini terdapat terdapat penjelasan penjelasan matematis matematis dari pertanyaa pertanyaan n ini. Kedua Lemma tersebut tersebut diturunkan dari teorema berikut: Teorema 3.1.1 Misalkan f Misalkan f adalah sebuah fungsi konveks pada interval buka I , jika x0 I maka berlaku
∈
≥ f ( f (x0 ) + f (x0 )(x )(x − x0 ) Pada fungsi konkav tanda ≥ diganti dengan ≤. Walupun alupun tidak terlalu terlalu sulit, f ( f (x)
bukti formal dari teorema ini memerlukan kalkulus dan tidak akan diberikan disini. disini. Ide yang mendasari mendasari pembentuk pembentukan an Teorema Teorema 1.8.1 adalah fakta bahwa bahwa pada fungsi konveks garis singgung pada suatu titik ( x0 , f ( f (x0 )) selalu berada dibawah dibawah grafik fungsi tersebut. tersebut. Sedangk Sedangkan an dari kalkulus kalkulus kita ketahu ketahuii bahwa bahwa persamaan garis singgung pada titik (x ( x0 , f ( f (x0 )) adalah y = f ( f (x0 ) + f (x0 )(x )(x x0 ). Berkutnya akan diperlihatkan penurunan lemma pada contoh 1.8.1 dengan teorema teorema ini. Pada contoh contoh 1.8.1 setelah diberikan diberikan syarat tanda samadenga samadengan n 1 terjadi jika a = b = c = 3 , jadi agar teorema bisa dipakai dengan mengawetkan x kasus samadengan kita pilih x0 = 13 pada teorema dengan fungsi f ( f (x) = 1− x. Fungsi f konveks di x (0, (0, 1) maka berdasarkan teorema 1.8.1
−
∈
f ( f (x) yang setara dengan
≥ f
− 1 3
1 3
x
9x − 1 ≥ 1−x 4
+ f
1 3
x
2
x+3) Sedangkan Sedangkan pada contoh 1.8.2 digunakan digunakan fungsi konkav konkav f ( f (x) = x2(+(3 −x)2 , dengan kasus samadengan terjadi jika j ika a = b = c = 1. Jadi kita gunakan teorema dengan x0 = 1 sehingga f ( f (x) f (1)(x (1)(x 1) + f (1) f (1)
≤
−
51
3.1. NORMALISASI NORMALISASI DAN SUBTITU SUBTITUSI SI yang setara dengan (x + 3)2 2x2 + (3 x)2
−
≤ 43 (x + 1)
Teorema 1.8.1 seringkali hanya digunakan untuk memperoleh lemma, sedangkan pembuktiannya dilakukan dengan cara aljabar biasa seperti pemfaktoran, pengkuadratan, dll. Perhatikan bahwa teorema 1.8.1 dapat gagal menghasilkan lemma jika f tidak konvek konvekss atau tidak konkav. konkav. Bahkan Bahkan pada contoh contoh berikut walaupun f konkav tapi teorema tersebut tidak membantu menyelesaikan soal. Contoh 3.1.3 Jika x,y, z > 0 buktikan x+
x + (x + y )(x )(x + z ) y +
y + (y + z )(y )(y + x) z +
Solusi Sisi kiri ketaksamaan diatas ekuivalen dengan x x+
x2 + xz + yx + yz
+
y y+
y 2 + xy + zy + xz
+
z (z + y )(z )(z + x)
≤1
z z+
+z 2 + yz + yz + xz
Karena ketaksamaan homogen maka WLOG xy + yz + xz = 1 jadi kita peroleh x √ + x + x2 + 1
y y+
y2
+1
+
z √ ≤1 z + +z 2 + 1
Untuk membuktikan ini kita lakukan subtitusi a = tan α , b = tan β dan c = tan γ dengan 0 < α,β, α,β, γ < π2 maka kita harus membuktikan: Jika tan α tan β + tan α tan γ + γ + tan β tan γ = 1 berlaku sin α sin β sin γ + + 1 + sin α 1 + sin β 1 + sin γ Lemma jika 0 < α, α, β,γ <
π 2
≤1
dan
tan α tan β + tan α tan γ + γ + tan β tan γ = 1 maka
π . 2 Bukti Lemma Pertimbangkan segitiga lancip ABC misalkan γ = mana kita peroleh α + β + γ =
tan ABC = karena BAC =
π 2
AC BC
dan
tan BAC =
− ABC maka kita peroleh identitas tan
− π 2
γ =
1 tan γ
BC AC
∠ABC
di-
52
BAB 3. PENJUMLAHAN PENJUMLAHAN SIMETRI SIMETRIK K DAN SIKLIS SIKLIS
Kemudian kalikan kedua ruas dengan tan
− π 2
γ
1 tan α+tan β
kita dapatkan
tan α + tan β tan α tan γ + γ + tan β tan γ tan α + tan β = 1 tan αβ = tan(α + β ) =
−
karena semua α,β,γ ada α,β,γ ada diinterval 0, π2 maka kita simpulkan π2 γ = α + β dan lemma terbukti. x Berikutnya karena fungsi f (x) = 1+x 1+x konkav menaik, dan fungsi g (x) = sin x konkav pada interval [0, [0 , π ] maka berdasarkan teorema 1.5.4 h(x) = (f g)(x )(x) = sin x [0, π]. Jadi berdasarkan berdasarkan ketaksam ketaksamaan aan Jensen dan Lemma 1+sin x konkav pada [0,
−
◦
sin α sin β sin γ + + 1 + sin α 1 + sin β 1 + sin γ
π
6 ≤ 13sin + sin π
=1
6
Pada contoh diatas walaupun fungsi φ(x) = x+√ xx2 +1 juga konkav pada interval terval [0 [ 0, ) namun apabila kita menggunakan ketaksamaan Jensen pada fungsi ini kita akan mendapatkan
∞
x √ + x + x2 + 1
y y+
y2 + 1
+
z √ ≤ x+y+z z + +z 2 + 1 3
sehingga selanjutnya kita harus membuktikan x+y+z x+y +z 3
+
x+y+z 2 3
+1
x+y+z +
x+y+z 3
+1
≤1
dibawah kondisi xy + yz + xz = 1. Tapi ketaks ketaksama amaan an ini salah salah karena karena jika jika x = y = 13 dan z = 43 maka 5 13 yang jelas salah. Kemudian Kemudian jika jika kita menggunak menggunakan an teorema teorema 1.8.1 mak makaa pembuktian pembuktian akan akan berlanjut dengan lemma berikut
≤√
t t + t2 + 1
√
−
√3 6
t
− 16 ≤ 0
Walaupun lemma ini benar, tapi apabila kita gunakan selanjutnya akan diharapkan 3 1 (x + y + z ) + 1 6 2 Sekali lagi ini salah karena jika x = y = 13 dan z = 43 maka dari ketaksamaan ini akan diperoleh 3 1 yang jelas salah. Hal ini berarti subtitusi trigonometri yang dilakukan pada solusi contoh 1.8.3 adalah sangat esensial (walaupun pada subbab 2 terdapat solusi yang lebih mudah). Pada soal - soal pertidaksamaan, teknik subtitusi sering dilakukan seperti pada Latihan Latihan 3 no 4. Berikut Berikut ini adalah soal - soal yang dapat diselesaik diselesaikan an dengan teknik subtitusi.
√
√ ≤
≤
53
3.1. NORMALISASI NORMALISASI DAN SUBTITU SUBTITUSI SI
Contoh 3.1.4 (IMO 1995) Misalkan a,b,c Misalkan a,b,c adalah bilangan riil positif dengan abc = 1 buktikan 1 a3 (b +
c)
+
1 1 + 3 + c) c ( a + b )
b3 (a
≥ 32
Solusi Ketaksamaan diatas tidak homogen tapi dengan melakukan subtitusi a = x1 , b = y1 dan c = z1 maka xyz = 1 dan kita peroleh ketaksamaan x2 y2 z2 + + y+z z+x x+y Ketaksamaan simetrik jadi WLOG x berdasarkan ketaksamaan Chebishev x2 y2 z2 + + y+z z+x x+y
≥
≥ 32
≥ y ≥ z maka
x+y+z 3
x y +z
≥
y x+z
≥
x y z + + y+z z+x x+y
z x+y
dan
Karena xyz = 1 maka berdasarkan AM-GM x+3y+z 1. Lal Lalu u ketak ketaksam samaan aan y 3 x z Nesbitt menyatakan y+z + z+x + x+y Perkalian keduanya akan melengkapi melengkapi 2 . Perkalian solusi.
≥
≥
Contoh 3.1.5 (AMM Januari 2006, Oleg Faynshteyn) Misalkan x Misalkan x, y dan 2 2 2 z adalah bilangan riil positif yang memenuhi x + y + z = 1 dan misalkan n adalah bilangan bulat positif. Tunjukkan bahwa x y z + + 2 n 2 n 1 x 1 y 1 z 2n
−
−
−
1+1/ 1+1/2n
(2n + 1) ≥ (2n 2n
Solusi Oleh Hendrata Dermawan Dermawan Misalkan
√
a = x( 2n 2n + 1)
√
√
b = y ( 2n 2n + 1)
c = z ( 2n 2n + 1)
Karena 0 < x,y,z < 1 maka 0 < a2n , b2n , c2n < 2n + 1. Sedangk Sedangkan an kondisi n 2 2 2 2 2 2 x + y + z = 1 berubah menjadi a + b + c = 2n + 1 dan ketaksamaan yang akan dibuktikan menjadi
√
a 2n + 1
b + 2n + 1
√2n + 1 n
c + 2n + 1
− b2n − c2n ≥ 2n √ dengan subtitusi a2 + b2 + c2 = 2n + 1, ketaksamaan diatas setara dengan − a2n
n
a 2n + 1
− a2n
+
b 2n + 1
− b2n
+
c 2n + 1
2
a ≥ − c2n
+ b 2 + c2 2n
atau
a 2n + 1
− a2n −
a2 2n
+
b 2n + 1
− b2n −
b2 2n
+
c 2n + 1
− c2n −
c2 2n
≥
0
54
BAB 3. PENJUMLAHAN PENJUMLAHAN SIMETRI SIMETRIK K DAN SIKLIS SIKLIS
Jadi untuk untuk menyele menyelesaik saikan an soal kita harus mem membukti buktika kan n ketaksam ketaksamaan aan diatas. diatas. Untuk membuktikannya, perhatikan bahwa ketaksamaan berikut benar a2 2n a + a2n+2 − (2n (2n + 1)a 1)a2 − ≥0 = − 2n 2n(2n (2n + 1 − a2n ) karena dengan AM-GM kita peroleh 2n 2 n a + a2n+2 ≥ (2n (2n + 1)a 1)a2 dan sebelumnya telah diperoleh 2n 2n + 1 − a2n ≥ 0. Dengan cara yang serupa b b2 2n b + b2n+2 − (2n (2n + 1)b 1)b2 − ≥0 = 2n + 1 − b2n 2n 2n(2n (2n + 1 − b2n ) a 2n + 1 a2n
dan
c 2n + 1
c2 2n c + c2n+2 − (2n (2n + 1)c 1)c2 − ≥0 = − c2n 2n 2n(2n (2n + 1 − c2n )
Tambahkan ambahkan ketiganya ketiganya diperoleh
a 2n + 1
− a2n −
a2 2n
+
b 2n + 1
− b2n −
b2 2n
+
c 2n + 1
− c2 n −
c2 2n
≥
seperti yang diinginkan.
Latihan 6 1. Jika x,y,z adalah bilangan riil positif, buktikan 1 1 1 + + x y z
9 9 9 ≥ 4(x + + 4(x + y) + z 4(x 4(x + z ) + y 4(y 4(y + z ) + x
2. Jika a , b dan c adalah panjang sisi - sisi segitiga buktikan a2
− b2 − c2 + bc + b2 − a2 − c2 + ac + c2 − b2 − a2 + ab ≤ a + b + c b+c
a+c
a+b
4
3. (Moldo (Moldov va 1999) 1999) Untuk a,b,c bilangan riil positif, buktikan ab bc ac + + c(a + c) a(a + b) b(b + c) 4. Jika 0
≥ a +a c + a +b b + c +c b
≤ α,β,γ < π2 dan sin2 α + sin2 β + sin2 γ = 1 maka buktikan tan α + tan β + tan γ ≥ cos2 α + cos2 β + cos2 γ
0
55
3.2.. SCHUR 3.2 SCHUR & MUIRH MUIRHEAD EAD
3.2 3.2
Sch Sc hur & Muir Muirhe head ad
Ketaksaamaan Schur dan Teorema Muirhead walaupun penggunaannya tidak begitu elegan tapi sangat ampuh untuk menyelesaikan menyelesaikan persoalan pertidaksamaan, kedua teorema tersebut sering digunakan dengan sebuah metode natural dalam Problem Solving yang disebut Brute Force yang berarti sesuai namanya yaitu memecahkan masalah dengan mencoba semua kemungkinan atau dalam pertidaksamaa tidaksamaan n ini berart b erartii melakuk melakukan an ekspansi ekspansi semua suku pada soal. Kita akan melihat bahwa disini sangat membantu unttuk menggunakan notasi cyc dan sym .
Teorema 3.2.1 (Ketaksamaan Schur) Misalkan x Misalkan x , y dan z dan z adalah bilangan riil positif. Untuk sebarang r > 0 berlaku xr (x
− y)(x )(x − z ) + y r (y − x)(y )(y − z ) + z r (z − y)(z )(z − x) ≥ 0.
Dengan kesamaan terjadi jika x = y = z atau jika dua variabel sama dan yang lainnya nol. Bukti WLOG x y z , maka xr y r (y z ) 0. Diperoleh
≥ ≥
≥ yr dan x − z ≥ y − z jadi xr (x − z) −
− ≥ (x − y)[x )[xr (x − z ) − y r (y − z )] + z r (z − y )(z )(z − x) ≥ 0
Q.E.D
Ketaksamaan Schur yang sering digunakan adalah kasus r = 1 yaitu yang berbentuk x3 + xyz x2 y
cyc
≥
sym
dan kasus r = 2 yang berbentuk
cyc
x4 + x2 yz
≥
x3 y
sym
Contoh Contoh beriku b erikutt memperlihat memperlihatk kan penggu p enggunaann naannya. ya. Meskipun Meskipun tidak elegan elegan ketaksamaan Schur sangat ampuh, dan merupakan langkah ”awal” jika teorema yang lain gagal. Contoh 3.2.1 (Iran 1996) Untuk x, y dan z bilangan riil positif berlaku (xy + yz + xz) xz )
1 1 1 + + 2 2 (x + y ) (x + z ) (y + z )2
≥
9 4
Solusi Semua Semua materi dari subbab subbab sebelumny sebelumnyaa gagal gagal.. Kita paksa dengan brute force, force, samakan penyebut diperoleh (xy + yz + xz) xz )
(x + y)2 (x + z )2
cyc
≥ 94 (x + y)2(y + z)2(x + z)2
56
BAB 3. PENJUMLAHAN PENJUMLAHAN SIMETRI SIMETRIK K DAN SIKLIS SIKLIS
setara dengan
−
(x2 + xy + yz + xz) xz )2
(xy+ xy +yz+ yz +xz) xz )
cyc
2 ≥ 94 ((x ((x + y + z )(xy )(xy + yz + xz) xz ) − xyz) xyz )
Ekspansi Ekspansi semua semua bentuk bentuk kuadrat kuadrat kita perole p eroleh h bentuk bentuk 4x5 y
x4 y 2
sym
− 3x3y3 + x4yz − 2x3y2z + x2y2z2 ≥ 0
untuk membuktikan ini kita pecah menjadi x4 yz
− 2x3y2z + x2y2z2 ≥ 0
sym
dan
4x5 y
sym
( )
∗
− x4y2 − 3x3y3 ≥ 0
( )
∗∗
Ketaksamaan (*) diperoleh dari ketaksamaan Schur untuk r = 1 yaitu x(x y)(x )(x z )+ y (y x)(y )(y z )+ z (z y )(z )(z x)
−
−
−
−
− ≥0⇔
−
1 3 1 x x2 y + xyz 2 2 sym
−
≥0
yang apabila dikalikan dengan 2xyz 2 xyz kita dapatkan ketaksamaan (*).Untuk mendapatkan ketaksaman (**), pertama tama kita lakukan AM-GM dengan cara berikut 16 y 8 1 1 1 4 4 x x5 y + y 5 x = 3 x5 y + y 5 x 4 = x4 y2 4 3 3 3 3 3 Yang mengahsilkan 3
x5 y +
sym
karena
sym
y5 x =
sym
≥ sym
y5 x
≥ ≥
x4 y 2
4
sym
x5 y maka
x5 y
sym
x4 y 2
(
sym
∗ ∗ ∗)
Setelah itu lakukan AM-GM dengan cara berikut
≥ ≥ ≥ ∗∗∗∗
x5 y + y5 x = 2 yang menghasilkan menghasilkan
1 2
x5 y + 2
1 2
x5
2
2
y5 x
4
4
x12 y12 = 2x 2 x3 y3 24
x3 y 3
sym
atau 3
x5
3
x3 y3
(
sym
tambahkan (***) dan (****) diperoleh (**). Q.E.D
)
57
3.2.. SCHUR 3.2 SCHUR & MUIRH MUIRHEAD EAD
Pada ketaksamaan Schur bentuk xr dapat diganti dengan fungsi tak-negatif yang menaik karena jika f adalah fungsi yang menaik dan selalu positif maka berlaku f ( f (x) f ( f (y ) f ( f (z ) untuk x y z dan f ( f (x)(x )(x z ) f ( f (y)(y )(y z ) 0 jadi (x y )[f )[f ((x)(x )(x z ) f (y )(y )(y z )] + f ( f (z )(z )(z y )(z )(z x) 0
≥ −
≥
≥ ≥ −
− −
− − − ≥ − − ≥
dan kita telah membuktikan Teorema 3.2.2 (Perluasan Ketaksamaan Schur) Jika f Jika f adalah fungsi yang selalu positif dan menaik di interval I , maka untuk setiap x,y,z I berlaku
∈ f (x)(x )(x − z )(x )(x − y ) + f ( f (y )(y )(y − x)(y )(y − z ) + f ( f (z )(z )(z − y )(z )(z − x) ≥ 0
Dapat dilihat bahwa semakin cekung grafik fungsi f maka teorema 19.2 akan semakin kuat, hal ini juga mengidentifikasikan bahwa untuk nilai r yang besar ketaksamaan Schur akan lebih tajam, dalam arti ekspresi disebelah kiri semakin mendekat ke 0. Setelah Ketaksamaan Schur diberikan, dapat dikatakan dikatakan bahwa semua materi pertidaksamaan yang esensial untuk tingkat IMO atau kompetisi yang serupa telah diberikan. Bagaimanapun juga dianjurkan bagi kita untuk mengenal lebih banyak banyak teorema teorema lagi. Materi Materi selanjutnya selanjutnya yaitu teorema teorema muirhead muirhead memang masih baru untuk digunakan di kompetisi, bahkan pada IMO 2005 yang lalu ada beberapa beberapa juri juri yang yang kurang kurang mengena mengenall teorem teoremaa ini. ini. Jadi Jadi dianju dianjurk rkan an bagi bagi yang akan menggunakannya menggunakannya pada kompetisi untuk membuktikannya membuktikannya terlebih dahulu dilembar jawaban. jawaban. Sebelum menyatakan menyatakan teorema Muirhead kita mulai dengan sebuah definisi yang juga akan berguna pada subbab berikutnya. Definisi Misalkan A = (a1 , a2 , , an ) dan B = (b1 , b2 , , bn) adalah pasangan terurut bilangan riil sehingga a1 a2 an dan b1 b2 bn . Kita katakan A majorize B ditulis A B jika dan hanya jika berlaku syarat berikut
···
a1 a1 + a2 a1 + a2 + a3 a1 + a2 +
···
· · · + an
Kita katakan A minorize B ditulis A berikut a1 a1 + a2 a1 + a2 + a3 a1 + a2 +
···
· · · + an
···
≥ ≥ ··· ≥ ≥ ≥ ≥
≥ ≥ · ·· ≥
b1 b1 + b2 b1 + b2 + b3
···
= b1 + b2 +
· · · + bn
B jika dan hanya jika berlaku syarat ≤ ≤ ≤
b1 b1 + b2 b1 + b2 + b3
···
= b1 + b2 +
· · · + bn.
58
BAB 3. PENJUMLAHAN PENJUMLAHAN SIMETRI SIMETRIK K DAN SIKLIS SIKLIS
Perlu ditekankan sekali lagi bahwa tanda samadengan pada definsi sangat penting. Definsi Definsi diatas merupakan merupakan definsi majorize dan minorize untuk pasangan terurut dengan syarat a1 a2 an dan b1 b2 bn , bagaimanabagaimanapun juga kita dapat mendefinisik mendefinisikan an majorize dan minorize untuk untuk sebarang sebarang pasangan terurut.
≥ ≥ · ·· ≥
≥ ≥ · ·· ≥
Definisi Misalkan A = (a1 , a2 , , an ) dan B = (b1 , b2 , , bn ) dan kemudian misalkan X = (x1 , x2 , , xn ) adalah permutasi dari A sehingga x1 x2 xn dan Y = (y1 , y2 , , yn) adalah permutasi dari B sehingga y1 y2 yn . Kita katakan A B jika dan hanya jika X Y , Y , secara secara serupa A B jika dan hanya jika X Y . Y .
··· ···
···≥ ···≥
···
···
≥ ≥ ≥ ≥
Teorema 3.2.3 Misalkan Misalkan vektor vektor atau pasanga pasangan n terurut terurut u = (a1 , a2 , dan v = (b1 , b2 , , bn ) jika u v maka terdapat matriks M
···
· · · λ1 λ2
λn−1 λn
dengan
n i=1 λi
= 1 dan λi
· · · , an )
λn λ1
· · · λ2 · · · λ3 · · · · · · ··· · · λn−2 · · · · · · λn−1 · · · λ1
≥ 0, sehingga MuT = v
Bukti dari teorema ini membutuhkan aljabar linear universitas tingkat lanjut 1 , dan tidak akan diberikan disini. Tapi untuk kasus n = 3 kita dapat mengunakan aljabar biasa. Teorema 3.2.4 (Muirhead) Misalkan pasangan terurut ( terurut (a1 , a2 , a3 ) maka untuk setiap bilangan riil x, y dan z berlaku
sym
xa1 ya2 z a3
≥
(b1, b2, b3)
xb1 yb2 z b3
sym
Bukti Perhatikan bahwa dengan Teorema 1.9.2 kita dapatkan terdapat λ1 + λ2 + λ3 = 1, λi 0 dengan
≥
a1 λ1 + a3 λ2 + λ3 a2 a2 λ1 + a1 λ2 + λ3 a3 a3 λ1 + a2 λ2 + λ3 a1 1
Lihat Inequalities Hardy-Littewood P¨ olya olya
= b1 = b2 = b3
59
3.2.. SCHUR 3.2 SCHUR & MUIRH MUIRHEAD EAD Perhatikan bahwa dengan AM-GM terboboti kita peroleh
xa1 y a2 z a3 = (λ1 + λ2 + λ3 )
sym
xa1 y a2 z a3
sym
≥
= λ1
xa1 y a2 z a3 + λ2
sym
= λ1
xa1 ya2 z a3 + λ3
sym a1 a2 a3
x y z
+ λ2
sym
xa1 ya2 z a3
sym a3 a1 a1
x y z
+ λ3
sym
xa2 ya3 z a1
sym
λ1 xa1 ya2 z a3 + λ2 xa3 ya1 z a2 + λ3 xa2 ya3 z a1
=
sym
x(a1 λ1 +a3 λ2 +λ3 a2 ) y (a2 λ1 +a1 λ2 +λ3 a3 ) z (a3 λ1 +a2 λ2 +λ3 a1 )
sym
xb1 y b2 z b3
=
sym
Teorema Muirhead juga berlaku untuk n variabel dengan bukti yang serupa. Contoh 3.2.2 (Ketaksamaan Carlson) Jika a,b,c Jika a,b,c adalah bilangan riil positif buktikan )(x + z )(y )(y + z ) xy + xz + yz 3 (x + y )(x 8 3
≥
Solusi Misalkan p = xy + xz + yz, yz , q = x + y + z dan r = xyz maka (a + b)(a )(a + c)(b )(b + c) = pq dan ketaksamaan menjadi
−r
− ≥ 3
pq
r
p 3
8
Pangkatkan 6 kita peroleh 27( p2 q 2
− 2 pqr + r2) ≥ 64 p3
Kemudian ekspansi semua kita peroleh
27 27x x4 yz + 27x 27x4 y2
sym
Dengan Muirhead (4, (4 , 1, 1)
− 30 30x x3 y 2 z − 5x3 y 3 − 19 19x x2 y 2 z 2 ≥ 0
(2, (2, 2, 2) maka 19 19x x4 yz ≥
sym
(4, (4, 2, 0)
(3, (3, 3, 0) maka
sym
≥
19 19x x2 y 2 z 2
sym
5x4 y 2
sym
5x3 y3
60 (4, (4, 1, 1)
BAB 3. PENJUMLAHAN PENJUMLAHAN SIMETRI SIMETRIK K DAN SIKLIS SIKLIS
(3, (3, 2, 1) maka
8x4 yz
sym
dan (4, (4, 2, 0)
≥
8x3 y2 z
22 22x x3 y 2 z
sym
(3, (3, 2, 1) maka
22 22x x4 yz
sym
≥
sym
tambahkan semua untuk menyelesaikan soal. Muirhead hanyalah aplikasi dari AM-GM, kita dapat menghindari pengunaan Muirhead dengan mengaplikasikan AM-GM beberapa kali, seperti pada bukti bukti ketaksam ketaksamaan aan (**) di solusi Iran 1996. Penggunaan Muirhead dan Schur biasanya dibarengi dengan homogenisasi. Yang dimaksud homogenisasi adalah kebalikan dari normalisasi. Jika sebelumnya kita mengubah ketaksamaan yang homogen menjadi non-homogen dengan menamba menambah h syarat, syarat, beikutnya beikutnya akan akan diperlihatk diperlihatkan an bahwa bahwa sebuah sebuah ketaksam ketaksamaan aan yang tak homogen dengan syarat dapat diubah menjadi yang homogen. Kesimpulannya membuat yang homogen menjadi tak homogen tidak selalu membuat masalah lebih mudah, dalam pengunaan ketaksamaan Schur dan Muirhead kita sering melakukan sebaliknya. Jika x,y,z adalah bilangan riil positif dengan xyz = 1 maka buktikan x3 + y 3 + z 3
≥ x2 + y2 + z2
Dapat dilihat sisi sebelah kanan berderajat 3 sedangkan yang sebelah kiri berderajat 2 , kita akan menyamakan derjatnya dengan menggunakan syarat xyz = 1, yaitu dengan mengkalikan sisi kanan dengan 3 xyz jadi diperoleh diperoleh
√
x3 + y3 + z 3
≥ x7/3y1/3z1/3 + x1/3y7/3z1/3 + x1/3y1/3z7/3
akhirnya dengan mengkalikan kedua ruas dengan 2 dan menggunakan notasi p eroleh h sym kita perole
≥ x3
sym
x7/3 y1/3 z 1/3
sym
7 1 1 yang benar karena dari teorema Muirhead (3, (3 , 0, 0) 3, 3, 3 . Berikut adalah sebuah contoh yang menggunakan homogenisasi
Contoh 3.2.3 (China TST 2006) Untuk x,y,z adalah adalah bilangan bilangan riil positif positif dengan x + y + z = 1 buktikan xy + xy + yz
√
√
yz + yz + xz
xz xz + xy
√
≤
√2 2
61
3.2.. SCHUR 3.2 SCHUR & MUIRH MUIRHEAD EAD
Solusi Dengan Dengan melakuk melakukan an homogenisas homogenisasii ketaksam ketaksamaan aan yang yang akan akan dibuktik dibuktikan an dapat ditulis sebagai sebagai berikut
cyc
xy xy + yz + xz
2(xy 2(xy + yz + xz) xz )2 (xy + yz)( yz )(x x + y + z )2
≤1
Dengan menggunakan ketaksamaan Jensen pada fungsi konkav f ( f (x) = peroleh
cyc
xy xy + yz + xz
xz) xz )2
2(xy 2(xy + yz + (xy + yz)( yz )(x x + y + z )2
Jadi tinggal dibuktikan
cyc
≤ cyc
2x(xy + yz + xz) xz ) (x + z )(x )(x + y + z )2
√x kita
2x(xy + yz + xz) xz ) (x + z )(x )(x + y + z )2
≤1
yang setara dengan x y z + + x+z x+y y+z
2
(x + y + z ) ≤ 2(xy 2(xy + yz + xz) xz )
perhatikan bahwa x y z (x + y + z )2 + + x+z x+y y+z 2(xy 2(xy + yz + xz) xz ) 2x(xy + yz + xz) xz ) 2y (xy + yz + xz) xz ) 2z (xy + yz + xz) xz ) + + (x + y + z )2 x+z x+y y+z 2 2 2 2 2 2 x z y x z y x +y +z + + x+z x+y y+z 2 2 2 2 x +y +z x2 z y2 x z2y x2 + y2 + z 2 + + 2 x+z x+y y+z 2 2 2 2 x +y +z x z y2 x z2 y x2 + y2 + z2 2 x+z x+y y+z x2 + y2 + z 2 x4 y4 z4 + + 2 x2 + xz xy + y2 yz + z 2
≤
⇔ ⇔ ⇔ ⇔ ⇔
≤
≤
− − −
≤
≤ ≤
−
Sedangkan dengan CS Engle dan ketaksamaan xy + yz + xz dapatkan x4 y4 z4 + + x2 + xz xy + y2 yz + z 2
≥
≤ x2 + y2 + z2 kita
(x2 + y2 + z 2 )2 (x2 + y2 + z 2 ) + xy + yz + xz
≥
x2 + y 2 + z 2 . 2
Contoh 3.2.4 (International Mathematical Olympiad 2000) Misalkan a,b,c Misalkan a,b,c adalah bilangan riil positif dengan abc = 1 buktikan
− − − ≤ a
1+
1 b
b
1+
1 c
c
1+
1 a
1
62
BAB 3. PENJUMLAHAN PENJUMLAHAN SIMETRI SIMETRIK K DAN SIKLIS SIKLIS
Solusi Kita akan membuat ketaksamaan diatas menjadi homogen, dengan cara melakukan subtitusi a = xy , b = yz dan c = xz akan diperoleh (x + y
− z)(y )(y + z − x)(x )(x + z − y) ≤ xyz
yang setara dengan
x3 + xyz
cyc
≥
x2 y
sym
ini merupakan ketaksamaan Schur untuk r = 1.
Latihan 7 1. (Japan (Japan 1997) 1997) Jika a,b,c adalah bilangan riil positif. Buktikan (b + c a)2 (c + a b)2 (a + b c)2 + + (b + c)2 + a2 (c + a)2 + b2 (a + b)2 + c2
−
−
−
≥ 35
2. (Bulgar (Bulgaria ia 1997) 1997) Jika a,b,c adalah bilangan riil positif dengan abc = 1. Buktikan 1 1 1 + + a+b+1 a+c+1 b+c+1
≤ 2 +1 a + 2 +1 b + 2 +1 c
3. (Vasile (Vasile Cartoaje) Jika a,b,c adalah bilangan riil dengan a2 + b2 + c2 = 1. Buktikan 1 1 1 9 + + 1 ab 1 bc 1 ac 2
−
−
− ≤
4. (LOGIKA 2006) Jika x,y,z adalah bilangan riil positif buktikan 1 1 1 27 + + + x y z x+y+z
36 36 36 ≥ 4(x + + 4(x + y ) + z 4(x 4(x + z ) + y 4(y 4(y + z ) + x
5. (Hojoo Lee) Jika a,b,c adalah bilangan bilangan riil positif dengan dengan ab + ac + bc = 1 buktikan 1 1 1 5 + + a+b a+c b+c 2
≥
Bab 4
Mixing Variabel dan Metode Kalkulus Sebuah metode yang akan kita pelajari berikutnya adalah sebuah metode yang biasanya dipakai apabila Scur atau Muirhead gagal. Metode ini simple tapi ampuh, yang yang dibutuhk dibutuhkan an adalah adalah kemampua kemampuan n mema memanipula nipulasi. si. Kita akan langsung mulai dengan sebuah contoh. Contoh 4.0.5 Jika a,b,c adalah adalah bilangan bilangan riil positif positif dengan abc = 1 maka buktikan a3 + b3 + c3 + 9 4(ab 4(ab + ac + bc) bc)
≥
Solusi Kita akan menggunakan metode Mixing variabel, idenya adalah dengan pertama - tama membuktikan bahwa ketaksamaan berlaku jika dua variabel kita samakan, dalam kasus ini karena terdapat syarat abc = 1, kita ganti dua buah variabel dengan variabel lain yang mempunyai hasil kali yang sama tapi hasil penjumlahannya lebih kecil. Misalkan f ( f (a,b,c) a,b,c) = a3 + b3 + c3 +9 4(ab 4(ab+ +ac+ ac+bc). bc). Dan WLOG a = min(a,b,c min(a,b,c), ), kemudian ganti variabel b dan c dengan t = bc, bc, perhatika perhatikan n bahwa bahwa dengan dengan penggantia penggantian n ini kita tetap memiliki memiliki at2 = abc = 1, keuntu keuntungann ngannya ya adalah kita memperoleh b + c 2t yang dapat menaikkan atau menurunkan fungsi f . f . Langkah selanjutnya adalah membuktikan
−
√
≥
f ( f (a,t,t) a,t,t)
≥0
Dimana
1 2 t7 t6
− 2 t5 + 5 t4 + 4 t3 + 3 t2 + 2 t + 1 (t − 1)2 yang jelas tidak negatif. Langkah selanjutnya adalah membuktikan f ( f (a,b,c) a,b,c) ≥ f ( f (a,t,t), a,t,t), yaitu membuktikan √ √ 3 f ( f (a,b,c) a,b,c) − f ( f (a,t,t) a,t,t) = b3 + c3 + 8a 8a bc − ( bc) bc) − 4a(b + c) ≥ 0 f ( f (a,t,t) a,t,t) = f 1/t2 ,t,t =
63
64
BAB 4. MIXING MIXING VARIABE VARIABEL L DAN METODE METODE KALKULUS KALKULUS
Yang setara dengan
√
√ 3 ≥ ( bc) bc) + 4a 4a(b + c) √ 3 √ ⇔ (b + c)3 − 3bc( bc(b + c) − 4a(b + c) ≥ 2( bc) bc) − 8a bc √ ⇔ (b + c)[(b )[(b + c)2 − 3bc − 4a] ≥ 2 bc( bc(bc − 4a) √ ⇔ (b + c)[b )[b2 + c2 − bc − 4a] ≥ 2 bc( bc(bc − 4a) √ Ketaksamaan yang terakhir benar karena dengan AM-GM b + c ≥ 2 bc dan juga b2 + c2 − bc − 4a ≥ bc − 4a juga dengan AM-GM. Jadi kita telah membuktikan Q.E.D f ( f (a,b,c) a,b,c) ≥ f ( f (a,t,t) a,t,t) ≥ 0 b3 + c3 + 8a 8a bc
Secara umum metode mixing variabel berkerja sebagai berikut:
• Pertama - tama bentuk fungsi f ( f (x1 , x2 , · · · , xn ). • Jika ketaksamaan simetrik maka untuk mempermudah asumsikan x1 ≥ x2 ≥ · · · ≥ xn , atau jika siklik asumsik asumsikan an a = min(a,b,c min(a,b,c). ). • Lalu buktikan f ( f (x1 , t, · · · , t) ≥ 0 atau f ( f (x1 , t, · · · , t) ≤ 0 dengan t adalah
fungsi dalam variabel lain, jika soal mempunyai terdapat syarat, maka t harus dibuat sedemika sedemikan n rupa sehingga syarat syarat tetap terpenuhi. terpenuhi. Contoh Contoh 3 jika syaratnya abc = 1 kita coba t = bc, bc, jika abcd = 1 coba bcd, bcd, jika c a + b + c = 1 coba b+ . Tentu saja ini tergantung masalahnya. 2
√
√
• Kemudian buktikan f ( f (x1 , x2 , · · · , xn ) ≥ f ( f (x1 , t, · · · , t) atau ≤. Cara diatas adalah mixing variabel yang biasa digunakan, kasus samadengan pada mixing variabel seperti diatas tergantung pada teorema yang digunakan pada waktu membuktikan f ( f (a,b,c) a,b,c) f (a,t,t) a,t,t) dan f ( f (a,t,t) a,t,t) 0 kemudian kasus tersebut tersebut ditambah ditambah lagi dengan pada saat nilai dua variabel variabel sama. sama. Kadangk Kadangkala ala dalam penggunaann penggunaannya ya Mixing Variabel dibantu dibantu oleh aplikasi aplikasi kalkulus alkulus seperti pada dua contoh berikut. berikut. Teorema eorema - teroema teroema berikut b erikut merumerupakan teorema kalkulus yang sudah familiar di Matematika SMA.
≥
≥
Teorema 4.0.5 (Teorema nilai Ekstrim) Misalkan f adalah sebuah fungsi yang kontinu di interval tertutup dan terbatas [a, b] maka f mencapai nilai minimum atau maximum di interval tersebut. Teorema 4.0.6 (Teorema titik Ekstrim) Misalkan f adalah sebuah fungsi yang kontinu di interval tertutup dan terbatas [a, b] maka nilai minimum atau maksimum tercapai pada tepat satu dari ketiga calon berikut
• Titik ujung yaitu a atau b • Titik stationer yaitu titik-titik x0 dimana f (x0) = 0 • Titik singular yaitu titik-titik titik-titik dimana f (x) tidak terdefinisi.
65 Contoh 4.0.6 Buktikan untuk setiap bilangan riil a,b,c berlaku
√4a21 + bc + √4b21+ ac + √4c21+ ab ≥ a + 4b + c Solusi Misalkan f ( f (a,b,c) a,b,c) =
√4a21 + bc + √4b21+ ac + √4c21+ ab − a + 4b + c
WLOG a + b + c = 1 dan a = min(a,b,c min(a,b,c)) kita akan menunjukkan
f a,
1
− a, 1 − a 2
2
= ϕ(a) =
√17 a2 +2 1 − 2 a + √4 − 64a + 2 a2 − 4
Dapat dicek bahwa lim a→0 ϕ(a) = 0 dan perhatikan bahwa
√4 − 64a + 2 a2 − 4 tidak mempunyai titik ekstrim di [0, [0 , 1], faktanya titik ekstrimnya adalah a = 32 yang terletak di luar [0, [0 , 1]. Jadi titik minimumnya terdapat dititik ujung yaitu a = 0 atau a = 1. Namun 2 17 a2 + 1 2 a
√
−
1 hanya mempunyai satu titik ekstrim a = 17 dan titik ekstrim ini adalah titik maksimum, sehingga titik minimumnya juga tercapai di titik ujung yaitu a = 0 atau a = 1. Ka Kare rena na ϕ(a) adalah adalah penjuml penjumlaha ahan n dari dari √ 4−6 4a+2 a2 4 dan √ 17 a22+1−2 a maka ϕ(a) juga juga mencap mencapai ai minim minimum um dititi dititik k ujung, ujung, dapat dapat dicek dicek bahwa ϕ(0) = 0 adalah nilai minimum ϕ(a) sedangkan lima→1 ϕ(a) = . c Selanjutnya akan ditunjukkan ditunjukkan f ( f (a,b,c) a,b,c) f ( f (a,t,t) a,t,t) dengan t = b+ 2 . Yang setara dengan menunjukkan menunjukkan
−
∞
≥
√ ≥
1
1
− √4a2 + t2 4a2 + bc
+
√4b21+ ac + √4c21+ ab − √4t22+ at ≥ 0
Ekspresi yang berada didalam tanda kurung jelas tidak negatif karena t2 = b+c 2 bc. bc. Sedangkan sisanya setara dengan 2
4t2 + at
≥ √ 1
4b2 +ac
2 + √ 4c12 +ab
Sisi sebelah kanan berbentuk HM (Harmonic ( Harmonic Mean ), ), jadi cukup bagi kita untuk menunjukkan sisi sebelah kiri lebih besar dari GM ( Geometrik Mean ) -nya. Ini setara dengan menunjukkan
≥ 4
b+c 2
2
+a
b+c 2
2
(4b (4b2 + ac)(4 ac)(4cc2 + ab) ab)
66
BAB 4. MIXING MIXING VARIABE VARIABEL L DAN METODE METODE KALKULUS KALKULUS
Ekspansi Ekspansi persamaan persamaan diatas, diatas, kemudian kemudian pertimbangk pertimbangkan an hasil ekspansi ekspansi sebagai sebagai polinomial berderajat 4 dalam b yang memiliki dua akar c. Jadi Jadi dengan dengan pemfaktoran ketaksamaan diatas setara dengan 2
a − c)2(b2 + c2 + 6bc − 3ab − 3bc) 6bc + bc) ≥ 0 4 yang benar karena dari asumsi a ≥ b ≥ c. (b
Contoh 4.0.7 (International Mathematics Olympiad 2006) Tentukan bilangan riil M terkecil sehingga ketaksamaan
|ab( ab(a2 − b2 ) + bc( bc(b2 − c2 ) + ca( ca(c2 − a2 )| ≤ M ( M (a2 + b2 + c2 )2 berlaku untuk sembarang bilangan riil a , b dan c Solusi Dari solusi contoh 1.5.5 kita peroleh
|ab( ab(a2 − b2 ) + bc( bc(b2 − c2 ) + ca( ca(c2 − a2 )|
= =
|(a − b)(b )(b − c)(a )(a − c)(a )(a + b + c)| |(a − b)(b )(b − c)(c )(c − a)(a )(a + b + c)|
Misalkan x = a b, y = b c, z = c a dan w = a + b + c maka x + y + z = 0 dan ketaksamaan pada soal dapat ditulis
−
−
−
|xyzw| ≤ M (x2 + y 2 + z 2 + w2 )2 9 Kemudian definisikan fungsi f ( f (x,y,z,w) x,y,z,w) =
|xyzw|
(x2 + y 2 + z 2 + w2 )2
Akan dicari nilai M sehingga f ( f (x,y,z,w) x,y,z,w)
≤ M 9
Kita akan mencari nilai maksimum dari f (x,y,z,w) x,y,z,w) kemudian dikalikan 9 untuk mendapatkan nilai M tersebut. tersebut. Sebelum menggunak menggunakan an Mixing Variabel, Variabel, p ertama - tama perhatikan bahwa diantara x, y dan z pasti ada dua diantaranya yang bertanda sama karena kondisi x + y + z = 0, jadi kasus dimana x = y , y = z atau x = z dijamin dapat terjadi, sehingga metode mixing variabel dapat senantiasa digunakan karena dalam menggunakan mixing variabel kita menyamakan dua buah variabel. Karena Karena fungsi f simetrik, simetrik, WLOG asumsikan asumsikan yang yang bertanda bertanda sama tersebut x+y adalah x dan y. Kemudian Kemudian misalkan misalkan t = 2 , karena x + y + z = 0 maka z = 2t. Jadi metode kita akan berlanjut dengan fungsi
−
f ( f (t,t, 2t, w) =
−
|2t3w|
(6t (6t2 + w2 )2
.
67 Dengan AM-GM kita dapatkan (6t (6t2 + w2 )2
= (2t (2t2 + 2t 2t2 + 2t 2t2 + w2 )2
√ √
(4 8t6 w2 )2 4
≥
= 32 2 wt3 Jadi
|wt3|
| | √
1 2 = 16 32 2
≤ √
(6t (6t2 + w2 )2
Dengan tanda samadengan terjadi jika dan hanya jika 2
2t = w
2
⇔ 2
dan x=y Subtitusikan b =
a+c 2
2
x+y 2
(a
− c)2 = (a ( a + b + c)2 2
⇔ b = a +2 c (a c)2 2
−
ke persamaan 2(a 2(a
=
= (a + b + c)2 diperoleh
− c)2 = 9(a 9( a + c)2
Selanjutnya kita dapat meniru langkah seperti pada contoh 1.5.5 dimana diperoleh kesamaan terjadi jika dan hanya jika (a,b,c ( a,b,c)) = 1 32 2, 1 + 32 2, 1 dan √ permutasinya. Jadi 9162 adalah adalah suatu dugaan nilai M yang beralasan. Sekarang tinggal dibuktikan
−√
f ( f (x,y,z,w) x,y,z,w)
≤ f (t,t, −2t, w)
dengan
t=
√
x+y 2
Yang setara dengan menunjukkan
− − | − − | ≤ | | ≤ | | | | ≤ xy( xy( x y)w 2 (x + y2 + z 2 + w2 )2
atau
xy 2 2 x + y + z 2 + w2
x+y 2 ( 2
2
2
2
x+y 2 2
z
y )w
+ z2 + w2
x+y 2 x+y 2 + z2 2
2
2
+ w2
Ini juga setara dengan 2
xy
x+y 2
S1
2
+
xy (z 2 + w2 ) S2
x+y x+y (x2 + y2 ) + (z 2 + w2 ) 2 2 S3
S4
68
BAB 4. MIXING MIXING VARIABE VARIABEL L DAN METODE METODE KALKULUS KALKULUS
Untuk membuktikan ketaksamaan ini, pertama-tama karena x dan y diasumsikan mempunyai tanda yang sama maka —x+y—=—x—+—y— dan —xy—=xy. Kita mulai dengan menunjukkan S 4 S 2 sebagai berikut
x+y 2
≥ |x| + |y| (z2 + w2) ≥ |xy|(z2 + w2) (z 2 + w2 ) = 2
jadi S 4 S 2 . Kemudian untuk untuk menunjukkan menunjukkan S 3 GM sebagai b erikut erikut
≥
|x + y| − √xy ≥ √xy
≥ S 1 kita mulai dengan AM|x + y| ≥ 2√xy
dan
Kalikan keduanya untuk mendapatkan
|x + y|2 − √xy|x + y| ≥ 2xy ⇔ ⇔ sehingga terbukti S 3
− √xy|x + y| ≥ 2xy √ x+y 2 x+y ≥ 2 xy 2 2
x2 + y2 + 2xy 2xy 2
2
(x + y )
≥ S 1, jadi S 1 + S 2 ≤ S 3 + S 4 dan pekerjaan kita selesai.
Mixing Mixing variabel variabel mempunyai mempunyai beberapa variasi. variasi. Pada contoh-co contoh-conto ntoh h diatas diatas kita melakukan mixing variabel variabel dengan menyamakan menyamakan beberapa variabel. Pada contoh berikut diperihatkan bahwa terdapat penggunaan lain. Contoh 4.0.8 (Hojoo Lee) Misalkan a,b,c adalah bilangan riil positif dengan ab + ac + bc = 1. Buktikan 1 1 1 + + a+b b+c a+c
≥ 52
Solusi Dari Pelatihan Pelatihan Tim IMO Cina (Dimodifikasi) (Dimodifikasi) Misalkan 1 1 1 5 f ( f (a,b,c) a,b,c) = + + a+b a+c b+c 2 WLOG a b c, kita akan menggunakan mixing varibel dengan cara berikut
−
≤ ≤
f ( f (a,b,c) a,b,c)
≥
1 f 0, a + b, a+b
perhatikan bahwa pasangan terurut (a,b,c ( a,b,c)) memenuhi ab + ac + bc = 1, begitu 1 juga pemilihan 0, a + b, a+ dikarenakan dua hal, yang pertama karena jika b
1 (a,b,c) a,b,c) diganti 0, a + b, a+ b maka kondisi ab + ac + bc = 1 tetap terjaga yaitu
0(a 0(a + b) + 0
1 a+b
+ (a ( a + b)
1 a+b
=1
dan yang kedua kedua karena kesamaan kesamaan pada soal terjadi jika salah satu variabel variabel bernilai 0 dan yang lain samadengan 1.
69
Seperti biasa langkah awal dimulai dengan membuktikan f 0, a + b, Yang setara dengan membuktikan 1 1 +a+b+ a+b a+b+ Misalkan x = kita perloleh
1 a+b
1 a+b
1 + (a ( a + b) a+b jadi sekarang tinggal membuktikan 1 5 untuk x 2 sedangkan dengan pemfaktoran kita dapatkan x+
≥
1 x+ x
5 2
⇔ − 1 2
5 2.
1 x
≥ 55 . Dengan AM-GM
≥2 x
≥2
− 2) ≥ 0. Langkah selanjutnya adalah membuktikan f ( f (a,b,c) a,b,c) ≥ f (0 f (0,, a + b, t) dengan t = 1 ≥
x
≥
≥ 52
+ (a ( a + b), maka kita harapkan x + x=
1 a+b
(x
a+b .
f (a,b,c) a,b,c) f (0 f (0,, a + b, t) 1 1 1 1 + + b+c a+c a+b+t t a + b + 2c 2c a + b + 2t 2t c2 + ab + ac + bc (a + b)t + t2 a + b + 2c 2c a + b + 2t 2t 2 2 c +1 1+t a + b + 2c 2c 1 + c2 a + b + 2t 2t 1 + t2 (a + b + 2c 2c)(1 + t2 ) (1 + c2 )(a )(a + b + 2t 2 t)
≥
⇔ ⇔ ⇔
≥
≥
≥
⇔ ≥ ⇔ ≥ ⇔ 2c + t2(a + b + 2c 2c) ≥ 2t + c2 (a + b + 2t 2 t) ⇔ 2(c 2(c − t) + t2 (a + b) − c2 (a + b) + 2t 2 t2 c − 2c2 t ≥ 0 ⇔ 2(c 2(c − t) + (t ( t2 − c2 )(a )(a + b) + 2tc 2 tc((t − c) ≥ 0 ⇔ (t − c)(−2 + (t(t + c)(a )(a + b) + 2tc 2 tc)) ≥ 0 Karena ac + bc ≤ 1 maka t − c ≥ 0 sedangkan sisanya diperoleh sebagai berikut (−2 + (t ( t + c)(a )(a + b) + 2tc 2 tc)) ≥ 0 ⇔ a 2+c b + c(a + b) − 1 ≥ 0 ⇔ a 2+c b + c(a + b) ≥ 1 ⇔ 2c + c(a + b)2 ≥ (a + b) ⇔ (2c (2c − a − b) + c(a + b)2 ≥ 0
70
BAB 4. MIXING MIXING VARIABE VARIABEL L DAN METODE METODE KALKULUS KALKULUS
yang benar karena dari asumusi c
≥ a dan c ≥ b mengakibatkan 2c 2 c ≥ a + b.
Sebuah solusi dapat menjadi sangat kreatif, beberapa diantaranya tidak dapat diklasifikasikan dalam bentuk metode-metode yang telah kita pelajari sebelumnya belumnya karena karena sangat sangat bervariasi bervariasi.. Pada Pada contohcontoh-con contoh toh diatas telah diperlihatkan bahwa suatu metode dapat menjadi lebih ampuh jika digunakan secara kreatif. Kadang-kadang peggunaan kalkulus diperlukan walaupun lebih bersifat teknis seperti mencari nilai maksimum atau minimum, atau mencari titik ekstrim. Pada contoh berikut kalkulus digunakan dengan cara yang lebih esensial Contoh 4.0.9 (MIT) Jika b > a > 0 buktikan
− a > √ab − ln a √ Solusi Pertimbangkan fungsi f ( f (t) = t − 1t − ln t, dimana t > 0. a+b b > 2 ln b
−
Dengan Dengan
menurunkan fungsi tersebut kita peroleh f (t) =
1 2
1
t1/2
+
1
t3/2
1 t
dengan mengaplikasikan AM-GM kita memperoleh 1 2
1 t1/2
+
1 t3/2
≥
1 t
⇐⇒ f (t) ≥ 0
dengan tanda samadengan tercapai jika dan hanya jika t = 1. Artin Artinya ya fungsi fungsi f (t) adalah fungsi naik sehingga untuk t> 1 =
⇒ f ( f (t) > f (1) f (1) = 0
misalkan t = ab , karena b > a maka t > 1 jadi f ( f (t) > 0
⇔ − ⇔
Sekarang Sekarang misalkan g (t) = ln t
b a b ln b
a > ln b
b a
− a > √ab − ln a
− 2tt+1−2 maka
1 g ( t) = − t
4 (t 1)2 = (t + 1)2 t(t + 1)2
−
jadi g (t) adalah fungsi naik sehingga untuk t> 1=
⇒ g(t) > g (1) = 0
71
4.1.. REFERE 4.1 REFERENSI NSI misalkan t =
b a
maka t > 1 sehingga g (t) > 0
⇔ ln
− b a
2b 2 a b a +1
−
>0
⇔ a +2 b > ln bb −− lna a . Latihan 7 1.(AMM Januari 2006, Aliyef Yakub) Misalkan a, b dan c adalah bilangan riil positif dengan a + b + c = 1. Buktikan 1 1 1 + + a b c
≥ 1 + 2548abc 48abc
1 Hint: Hint: Misalkan Misalkan a = min( min(a,b,c a,b,c)) maka 0 < a Lalu Lalu gunak gunakan an fung fungsi si 3. 1 1 1 f ( f (a,b,c) a,b,c) = a + b + c + 48(ab 48(ab + ac + bc) bc) 25. 2.(Fajar Yuliawan & Dimas Yusuf) Temukan bilangan riil terbesar k sehingga hingga untuk untuk setiap setiap bilangan bilangan riil positif p ositif a,b,c a,b,c dengan abc = 1 berlaku
−
a2 + b2 + c2 + 3k 3k
≤
≥ (k + 1)(a 1)(a + b + c)
3.(Romania 3.(Romania 2006) Untuk bilangan riil positif a, b dan c dengan a + b + c = 3 buktikan 1 1 1 + 2 + 2 a2 + b2 + c2 2 a b c
≥
4.1 4.1
Refe Refere rens nsii
www.mathlinks.ro www.olimpiade.org Hojoo, Hojoo, Lee. Topics In Inequality. Mildorf. Inequalities. Inequalities. Kiran, Keldaya. A > B .